Tải bản đầy đủ (.docx) (147 trang)

cac dinh ly hinh hoc so cap

Bạn đang xem bản rút gọn của tài liệu. Xem và tải ngay bản đầy đủ của tài liệu tại đây (3.77 MB, 147 trang )

<span class='text_page_counter'>(1)</span>Bộ giáo dục và đào tạo ViÖn to¸n häc. MỘT SỐ KIẾN THỨC VỀ HÌNH OLYMPIAD. Ngêi ViÕt : Ths Lª §×nh HËu Chøc vô : Gi¸o viªn §¬n vÞ c«ng t¸c : Trêng THPT Quan Ho¸, tØnh thanh ho¸. Thanh Ho¸, th¸ng 01-2010.

<span class='text_page_counter'>(2)</span> MỘT SỐ KIẾN THỨC VỀ HÌNH OLYMPIAD (Mathscope.org) Những kiến thức sau đây gồm một số kiến thức cơ sở để khám phá hình học olympiad hoặc là những kết quả đẹp nổi tiếng :hornytoro:.Bài viết này được soạn ra nhằm đáp ứng nhu cầu tra cứu ,học hỏi của nhiều bạn đọc. Nó sẽ cần sự chung tay của nhiều thành viên !. Đầu tiên mình sẽ giới thiệu mục lục và nếu ai biết phần kiến thức ấy thì có thể post lên , nhưng để đảm bảo cho tính hệ thống , chặt chẽ và dễ theo dõi của bài viết ,mình xin nêu một số quy ước như sau: 1) Mỗi bài viết đều phải vẽ hình minh họa. 2)Mỗi bài viết chỉ đề cập đến 1 đề mục kiến thức. 3) Phải đảm bảo thứ tự nêu trong mục lục. 4)Chúng tôi chỉ giữ lại những trao đổi có ích kể từ sau khi hoàn thành mục lục, điều đó có nghĩa là những trao đổi chen giữa không bị xóa lúc này nhưng sẽ bị xóa khi mục lục được hoàn tất.. Bây giờ sẽ là nội dung chính A/ MỤC LỤC I/ Một số định nghĩa ,định lí , điểm và đường đặc biệt không duy nhất : I.1)Định lí Menelaus I.2)Mở rộng định lí Menelaus theo diện tích I.3)Định lí Menelaus cho tứ giác I.4)Định lí Ceva I.5)Định lí Ceva dạng sin I.6)Định lí Desargues I.7)Định lí Pappus I.8)Một trường hợp đặc biệt của định lí Pappus qua góc nhìn hình xạ ảnh. I.9)Đẳng thức Ptolemy I.10)Bất đẳng thức Ptolemy I.11)Định lí Pascal I.12)Định lí Brianchon I.13)Định lí Miquel I.14)Công thức Carnot I.15)Định lí Carnot I.16)Định lí Brokard I.17)Định lí Euler về khoảng cách giữa tâm 2 đường tròn nội, ngoại tiếp của tam giác I.18)Định lí Euler về khoảng cách giữa tâm 2 đường tròn nội, ngoại tiếp của tứ giác (Định lí Fuss) I.19)Định lí Casey I.20)Định lí Stewart I.21)Định lí Lyness I.22)Định lí Lyness mở rộng (Bổ đề Sawayama).

<span class='text_page_counter'>(3)</span> I.23)Định lí Thébault I.24)Công thức Jacobi liên quan đến tâm tỉ cự,định lí Lebnitz I.25)Định lí Newton cho tứ giác ngoại tiếp I.26)Định lí Breichneider I.27)Định lí con nhím I.28)Định lí Gergonne -Euler I.29)Định lí Peletier I.30)Định lí Miobiut I.31)Định lí Viviani I.32)Công thức Lagrange mở rộng I.33) Đường thẳng Simson I.34)Đường thẳng Steiner I.35) Điểm Anti-Steiner (Định lí Collings) I.36)Định lí Napoleon I.37)Định lí Morley I.38)Định lí con bướm với đường tròn I.39)Định lí con bướm với cặp đường thẳng I.40)Điểm Blaikie I.41)Định lí chùm đường thẳng đồng quy I.42)Đường tròn Apollonius I.43)Định lí Blanchet I.44)Định lí Blanchet mở rộng I.45) Định lí Jacobi I.46) Định lí Kiepert I.47)Định lí Kariya I.48)Cực trực giao I.49)Khái niệm tam giác hình chiếu ,công thức Euler về diện tích tam giác hình chiếu I.50)Khái niệm hai điểm đẳng giác I.51)Khái niệm tứ giác toàn phần. I.52)Đường thẳng Droz-Farny I.53) Đường tròn Droz-Farny I.54)Định lí Van Aubel về tứ giác và các hình vuông dựng trên cạnh I.55)Hệ thức Van Aubel I.56)Định lí Pithot I.57)Định lí Johnson I.58) Định lí Eyeball I.59) Bổ đề Haruki I.60)Bài toán Langley I.61)Định lí Paul Yiu về đường tròn bàng tiếp. I.62)Định lí Maxwell I.63)Định lí Brahmagupta về tứ giác nội tiếp có hai đường chéo vuông góc. I.64)Định lí Schooten I.65)Định lí Bottema I.66)Định lí Pompeiu I.67)Định lí Zaslavsky I.68)Định lí Archimedes.

<span class='text_page_counter'>(4)</span> I.69) Định lí Urquhart I.70)Định lí Mairon Walters I.71)Định lí Poncelet về bán kính đường tròn nội tiếp,bàng tiếp trong tam giác vuông. I.72)Định lí Hansen I.73)Định lí Steinbart suy rộng I.74)Định lí Monge & d'Alembert I I.75)Định lí Monge & d'Alembert II I.76)Định lí Steiner về bán kính các đường tròn. I.77)Định lí Bellavitis I.78)Định lí Feuer bach-Luchterhand:. II/Một số điểm và đường đặc biệt được xác định duy nhất với tam giác và tứ giác,tứ điểm: Ở đây nếu không giải thích gì thêm thì yếu tố được hiểu là trong tam giác.. II.1) Đường thẳng Euler của tam giác II.2)Đường tròn và tâm Euler II.3)Đường đối trung, điểm Lemoine II.4)Điểm Gergone,điểm Nobb, đường thẳng Gergone II.5)Điểm Nagel II.6)Điểm Brocard II.7)Điểm Schiffler II.8)Điểm Feuerbach II.9)Điểm Kosnita II.10)Điểm Musselman,định lí Paul Yiu về điểm Musselman II.11)Khái niệm vòng cực của tam giác. II.12)Điểm Gibert II.13)Trục Lemoine II.14)Tâm Morley II.15) Tâm Spieker và đường thẳng Nagel II.16)Hai điểm Fermat II.17)Điểm Parry reflection. II.18)Đường tròn Taylor ,tâm Taylor II.19)Điểm Bevan II.20)Điểm Vecten II.21)Điểm Mittenpunkt II.22)Điểm Napoleon.

<span class='text_page_counter'>(5)</span> II.23)Đường tròn Adam II.24)Tam giác Fuhrmann ,đường tròn Fuhrmann II.25)Hình luc giác và đường tròn Lemoine thứ nhất II.26)Hình lục giác và đường tròn Lemoine thứ hai II.27)Điểm Euler của Tứ giác nội tiếp II.28)Đường thẳng Steiner của tứ giác toàn phần II.29)Đường thẳng Gauss của tứ giác toàn phần. II.30) Điểm Miquel của tứ giác toàn phần II.31)Đường tròn Miquel của tứ giác toàn phần II.32)Hình bình hành Varignon của tứ giác . II.33)Điểm Poncelet của tứ giác.. III/Một số mảng kiến thức quan trọng. III.1)Tỉ số kép, phép chiếu xuyên tâm III.2)Hàng điểm điều hòa và một số hệ thức liên quan , III.3)Chùm điều hòa, tứ giác điều hòa III.4)Góc giữa đường thẳng và đường tròn, giữa hai đường tròn, đường tròn trực giao III.5) Cực và đối cực IV/Một số định lí không chứng minh Ở đây sẽ giới thiệu một số định lí rất hay và dễ hiểu ( nhưng cách chứng minh mà mình biết là phức tạp ) tuy nhiên rất vui nếu ai đó sẽ giới thiệu những chứng minh của nó:hornytoro:. IV.1) Định lí Aiyer IV.2)Đường tròn Lester IV.3)Tâm Eppstein IV.4)Đường tròn Neuberg-Mineur của tứ giác IV.5)Paracevian perspector. B/MỘT SỐ KHÁI NIỆM,ĐỊNH LÍ. I.1)Định lí Menelaus.

<span class='text_page_counter'>(6)</span> Định lí: Cho tam giác ABC và 3 điểm M,N,P lần lượt thuộc BC,CA,AB. Khi đó M,N,P thẳng hàng khi và chỉ khi: (1) Chứng minh:. a)Khi M,N,P thẳng hàng. Trên MN lấy 1 điểm Q sao cho AQ//BC Theo Thales ;. Từ đó dễ có đẳng thức (1)trên. b)Ngược lại ,khi có (1): Giả sử PN cắt BC tại M'. Theo phần trước ta có:. Kết hợp với (1) suy ra Do đó M trùng M' tức là M,N,P thẳng hàng..

<span class='text_page_counter'>(7)</span> Vậy ta có điều phải chứng minh. (Xem them : eeg-11.bdf; ge_G1.bdf; 6-concur-solns.bdf) I.2)Mở rộng định lí Menelaus theo diện tích Định lí:Cho tam giác ABC và 3 điểm M,N,P lần lượt nằm trên BC,CA,AB.Khi đó ta có:. Chứng minh :(thamtuhoctro post). Gọi Ta có:. mặt khác :. tương tự:. là vector chỉ phương của.

<span class='text_page_counter'>(8)</span> Ta suy ra:. I.3)Định lí Menelaus cho tứ giác: Định lí:Cho tứ giác ABCD và một đường thẳng d cắt AB,BC,CD,DA lần lượt ở M,N,P,Q. Khi đó ta có:. Chứng minh: Ta sẽ làm giống cách chứng minh ở tam giác Trên d lấy hai điểm I,J sao cho AI//BJ//CD Theo Thales ta có:.

<span class='text_page_counter'>(9)</span> Từ đó dễ có điều cần chứng minh. *Chú ý 1)Khi áp dụng cho tứ giác ,định lí Menelaus chỉ phát biểu dạng thuận bởi dạng đảo nói chung không đúng! 2) Các bạn thử suy nghĩ xem với dạng thuận như thế này thì có thể mở rộng cho đa giác được không? -Một vấn đề khá thú vị. I.4) Định lý Ceva Định lý: Cho tam giác ABC.Gọi E, F, G là ba điểm tương ứng nằm trên BC, CA, AB. Ba đường thẳng AE, BF, CG cắt nhau tại một điểm O khi và chỉ khi:. Chứng minh: Phần thuận: Giả sử ba đường thẳng AE, BF, CG cắt nhau tại một điểm O. TỪ A và C, kẻ các đường song song với BF, chúng lần lượt cắt CG và AE tại K, I tương ứng. Ta có:. và. (Sử dụng định lý Thales). . Các cặp tam giác đồng dạng IEC và OEB, AKG và BOG : Do đó: Phần đảo:. và.

<span class='text_page_counter'>(10)</span> Giả sử ta có: Qua giao điểm của các đường thẳng AE và BF, kẻ đường thẳng. với. nằm trên cạnh AB. Khi. đó, theo chứng minh phần thuận: Suy ra. , hay. , ta có điều phải chứng minh. I.5) Định lý Ceva sin Định lý: Gọi E, F, G là ba điểm tương ứng nằm trên các đường thẳng BC, CA, AB của tam giác ABC. Ba đường thẳng AE, BF, CG cắt nhau tại một điểm O khi và chỉ khi:. Chứng minh: Phần thuận: Giả sử AE, BF, CG đồng quy tại O. Khi đó hai tam giác ABE và ACE có cùng chiều cao hạ từ đỉnh A. Tương tự Và Nhân từng vế ba đẳng thức trên được: Từ đó suy ra đpcm. Phần đảo: CM tương tự phần đảo ở mục 4.. (Theo định lý Ceva).

<span class='text_page_counter'>(11)</span> I.6) Định lý Desargues Định lý: Cho tam giác ABC và tam giác A'B'C'. Khi đó AA', BB', CC' đồng quy khi và chỉ khi các giao điểm của BC và B'C', CA và C'A', AB và A'B' thẳng hàng.. Chứng minh: Gọi X, Y, Z là lần lượt là các giao điểm của các cặp cạnh BC và B’C’, CA và C’A’, AB và A’B’ . Phần thuận: Giả sử các đường thẳng AA’, BB’, CC’ đồng quy tại S. Ta chứng minh X, Y, Z thẳng hàng. Áp dụng định lí Menelaus cho tam giác SBC với cát tuyến XB'C' ta có: hay Tương tự, ta có: và Nhân từng vế các đẳng thức trên lại với nhau, và theo định lí Menelaus suy ra X, Y, Z thẳng hàng..

<span class='text_page_counter'>(12)</span> Phần đảo: Giả sử các điểm X, Y, Z thẳng hàng. Ta chứng minh các đường thẳng AA’, BB’, CC’ đồng quy. Gọi S là giao điểm của AA’ và BB’. SC cắt đường thẳng AC’ tại C”. Xét 2 tam giác ABC và A’B’C” có các đường nối các đỉnh tương ứng đồng quy, do đó theo phần thuận giao điểm của các cạnh tương ứng cũng đồng quy. Ta thấy AB cắt A’B’ tại Z, AC cắt A’C” tại Y (do A’, C’, C” thẳng hàng), suy ra giao điểm X’ của BC và B’C” phải thuộc YZ. Tức là X’ là giao của YZ và BC nên X’ trùng với X. Suy ra C” trùng với C’, hay AA’, BB’, CC’ đồng quy.. I.7)Định lí Pappus Định lí: Cho ba điểm A,B,C nằm trên đường thẳng a, X,Y,Z nằm trên đường thẳng b.Gọi M,N,P lần lượt là giao điểm của các cặp đường thẳng (AY,BX) ,(AZ,CX),CY,BZ). Khi đó M,N,P thẳng hàng. Chứng minh:. Định lí này có một cách chứng minh dùng Menelaus ,nếu có điều kiện mình sẽ post lên,còn sau đây là một cách dựa trên kiến thức cơ sở về tỉ số kép và phép chiếu xuyên tâm. Ta có bổ đề sau được chứng minh dễ dàng nhờ những hiểu biết ban đầu về tỉ số kép và phép chiếu xuyên tâm: Bổ đề: Cho góc xOy và các điểm A,B,C thuộc Ox; D,E,F thuộc Oy. Khi đó AD,BE,CF đồng quy khi và chỉ khi: (OABC) =(ODEF) . Bổ đề trên bạn đọc tự chứng minh, bây giờ ta sẽ trở lại bài toán. Kí hiệu là phép chiếu xuyên tâm E..

<span class='text_page_counter'>(13)</span> Gọi T,Q lần lượt là giao điểm của BX và AZ; CX và BZ. Sử dụng bổ đề trên thì ta sẽ cần chứng minh: (BTMX) =(BZPQ) +)Trường hợp a//b bạn đọc hãy chứng minh nhờ Thales +)Khi a không song song với b.Gọi S là giao của a và b. Ta thấy: Với : Với Từ đó suy ra điều cần chứng minh.. I.8)Một trường hợp đặc biệt của định lí Pappus qua góc nhìn hình xạ ảnh. Ở phần này chúng tôi chỉ dùng hình xạ ảnh để dẫn dắt đến kết quả còn nội dung định lí và cách chứng minh thì hoàn toàn phù hợp với kiến thức hình THCS! Ta có kết quả sau liên quan đến hình xạ ảnh: Các đường thẳng song song với nhau thì gặp nhau tại một điểm ở vô cực và ngược lại . Vận dụng vào định lí Pappus ở trên , cho các điểm A,B,C ra vô cực thì theo kết quả về hình xạ ảnh ta có YM//ZN ( Vì YM,ZN cùng đi qua một điểm (A) ở vô cực )Tương tự thì :XN//YP,XM//ZP. Và khi ấy M,N,P vẫn thẳng hàng. Ta phát biểu lại được một định lí đơn giản và hữu dụng sau đây: Định lí:Trên mặt phẳng cho ba điểm X,Y,Z thẳng hàng và ba điểm M,N,P thỏa mãn XN//YP,YM//ZN,XM//ZP. Khi đó ta cũng có M,N,P thẳng hàng. Chứng minh:.

<span class='text_page_counter'>(14)</span> Trường hợp MP//XYZ thì đơn giản,bạn đọc tự chứng minh. Ta sẽ xét khi MP không song song với XYZ. Gọi S là giao điểm của MP với XYZ. Đường thẳng qua X song song với YP cắt MP ở N'. Bài toán sẽ được gải quyết nếu ta chứng minh được rằng ZN' // YM (Vì khi ấy N' trùng N). Thật vậy,chú ý YP//XN', ZP//XM nên theo Thales ta có: Đến đây theo Thales đảo ta suy ra ZN' //YM. Chứng minh được hoàn tất.!. I.9)Đẳng thức Ptolemy Định lí Với tứ giác nội tiếp ABCD thì: AB.CD+AD.BC=AC.BD Chứng minh:.

<span class='text_page_counter'>(15)</span> Lấy điểm E thuộc AC sao cho đồng dạng Tương tự. đồng dạng. I.10) Bất đẳng thức Ptolemy Định lý: Cho tứ giác ABCD. Khi đó có. Chứng minh: Lấy E nằm trong tứ giác ABCD sao cho và.

<span class='text_page_counter'>(16)</span> Khi đó Hơn nữa Vậy ta có. ~. hay ~. . hay (đpcm).. KHÁM PHÁ ĐỊNH LÍ PTÔ-LÊ-MÊ I.. tác giả:Zai zai Mở đầu: Hình học là một trong những lĩnh vực toán học mang lại cho người yêu toán nhiều điều thú vị nhất và khó khăn nhất. Nó đòi hỏi ta phải có những suy nghĩ sáng tạo và tinh tế. Trong lĩnh vực này cũng xuất hiện ko ít những định lí, phương pháp nhằm nâng cao tính hiệu quả trong quá trình giải quyết các bài toán, giúp ta chinh phục những đỉnh núi ngồ ghề và hiểm trở . Trong bài viết này zaizai xin giới thiệu đến các bạn một vài điều cơ bản nhất về định lí Ptô-lê-mê trong việc chứng minh các đặc tính của hình học phẳng. Dù đã rất cố gắng nhưng bài viết sẽ không thể tránh khỏi những thiếu xót mong rằng các bạn sẽ cùng zaizai bổ sung và phát triển nó. II, Nội dung - Lí thuyết: 1. Đẳng thức Ptô-lê-mê: Cho tứ giác nội tiếp đường tròn. Hình minh họa (hình 1). Chứng minh:. . Khi đó:.

<span class='text_page_counter'>(17)</span> Lấy. thuộc đường chéo. sao cho. Khi đó xét và Nên đồng dạng với Do đó ta có:. có:. . Lại có: Suy ra Từ. và. và. nên. hay. suy ra:. Vậy đẳng thức Ptô-lê-mê được chứng minh. 2, Bất đẳng thức Ptô-lê-mê: Đây có thể coi là định lí Ptô-mê-lê mở rộng bởi vì nó không giới hạn trong lớp tứ giác nội tiếp . Định lí: Cho tứ giác. . Khi đó:. Hình minh họa (hình 2). Chứng minh: Trong. lấy điểm M sao cho:. Dễ dàng chứng minh: Cũng từ kết luận trên suy ra: Áp dụng bất đẳng thức trong tam giác và các điều trên ta có: Vậy định lí Ptô-lê-mê mở rộng đã được chứng minh. 3, Định lí Ptô-lê-mê tổng quát: Trong mặt phẳng định hướng cho đa giác điểm thuộc cung (Không chứa Khi đó: .. nội tiếp đường tròn ). . M là một.

<span class='text_page_counter'>(18)</span> Trong đó: Đây là một định lí ko dễ dàng chứng minh được bằng kiến thức hình học THCS. Các bạn có thể tham khảo phép chứng minh trong bài viết Định lí Ptô-lê-mê tổng quát của Tiến sĩ Nguyễn Minh Hà, ĐHSP , Hà Nội thuộc Tuyển tập 5 năm Tạp chí toán học và tuổi trẻ. III, Ứng dụng của định lí Ptô-lê-mê trong việc chứng minh các đặc tính hình học: 1, Chứng minh quan hệ giữa các đại lượng hình học: Mở đầu cho phần này chúng ta sẽ đến với 1 ví dụ điển hình và cơ bản về việc ứng dụng định lí Ptô-lê-mê. Bài toán 1: Cho tam giác đều có các cạnh bằng Trên lấy điểm di động, trên tia đối của tia lấy điểm di động sao cho . Gọi là giao điểm của và . Chứng minh rằng: ( Đề thi vào trường THPT chuyên Lê Quí Đôn, thị xã Đông Hà, tỉnh Quảng Trị, năm học 2005-2006) Hình minh họa (hình 3). Chứng minh: Từ giả thiết Xét và. suy ra có:. Lại có Từ: Suy ra tứ giác nội tiếp được đường tròn. Áp dụng định lí Ptô-lê-mê cho tứ giác nội tiếp và giả thiết có: (đpcm). ta. Đây là 1 bài toán khá dễ và tất nhiên cách giải này ko được đơn giản lắm.Vì nếu muốn sử dụng đẳng thức Ptô-lê-mê trong 1 kì thi thì có lẽ phải chứng minh nó dưới dạng bổ đề. Nhưng điều chú ý ở đây là ta chẳng cần phải suy nghĩ nhiều khi dùng cách trên trong khi đó nếu dùng cách khác thì lời giải có khi lại ko mang vẻ tường minh..

<span class='text_page_counter'>(19)</span> Bài toán 2: Tam giác vuông có . Gọi là một điểm trên cạnh là một điểm trên cạnh kéo dài về phía điểm sao cho . Gọi là một điểm trên cạnh sao cho nằm trên một đường tròn. là giao điểm thứ hai của với đường tròn ngoại tiếp . Chứng minh rằng: (Đề thi chọn đội tuyển Hồng Kông tham dự IMO 2000, HongKong TST 2000) Hình minh họa: (hinh 4). Chứng minh: Xét các tứ giác nội tiếp. và. ta có:. (cùng chắn các cung tròn) Mặt khác Xét (do. và. có: ). (do ) Áp dụng định lí Ptô-lê-mê cho tứ giác nội tiếp Từ. ta có:. suy ra: (đpcm). Có thể thấy rằng bài 1 là tư tưởng đơn giản để ta xây dựng cách giải của bài 2. Tức là dựa vào các đại lượng trong tam giác bằng nhau theo giả thiết ta sử dụng tam giác đồng dạng để suy ra các tỉ số liên quan và sử dụng phép thế để suy ra điều phải chứng minh. Cách làm này tỏ ra khá là hiệu quả và minh họa rõ ràng qua 2 ví dụ mà zaizai đã nêu ở trên. Để làm rõ hơn phương pháp chúng ta sẽ cùng nhau đến với việc chứng minh 1 định lí bằng chính Ptô-lê-mê. Bài toán 3: ( Định lí Carnot) Cho tam giác nhọn nội tiếp trong đường tròn và ngoại tiếp đường tròn lần lượt là khoảng cách từ tới các cạnh tam giác. Chứng minh rằng:. Gọi.

<span class='text_page_counter'>(20)</span> Hình minh họa (hinh 5). Chứng minh: Gọi lần lượt là trung điểm của Tứ giác. . Giả sử. nội tiếp, theo đẳng thức Ptô-lê-mê ta có:. Do đó: Tương tự ta cũng có :. Mặt khác: Từ. ta có:. Đây là 1 định lí khá là quen thuộc và cách chứng minh khá đơn giản. Ứng dụng của định lí này như đã nói là dùng nhiều trong tính toán các đại lượng trong tam giác. Đối với trường hợp tam giác đó không nhọn thì cách phát biểu của định lí cũng có sư thay đổi. 2, Chứng minh các đặc tính hình học: Bài toán 1: Cho tam giác tiếp xúc với đường tròn giữa của cung Hình minh họa(hinh 6). nội tiếp trong đường tròn và tại cắt nhau ở . Chứng minh rằng. . Các đường thẳng đi qua điểm chính.

<span class='text_page_counter'>(21)</span> Chứng minh: Gọi giao điểm của Xét. và. với đường tròn là có:. . Nối. .. chung. Tương tự ta cũng có Mặt khác ( do là 2 tiếp tuyến của đường tròn cắt nhau) Nên từ Áp dụng định lí Ptô-lê-mê cho tứ giác nội tiếp Từ Vậy ta có điều phải chứng minh.. ta có: .. Đây có lẽ là một trong những lời giải khá là ngắn và ấn tượng của bài này.Chỉ cần qua vài quá trình tìm kiếm các cặp tam giác đồng dạng ta đã dễ dàng đi đến kết luận của bài toán. Tư tưởng ban đầu khi làm bài toán này chính là dựa vào lí thuyết trong cùng một đường tròn hai dây bằng nhau căng hai cung bằng nhau. Do có liên quan đến các đại lượng trong tứ giác nội tiếp nên việc chứng minh rất dễ dàng. Bài toán 2: Cho tam giác ABC có I là tâm đường tròn nội tiếp, O là tâm đường tròn ngoại tiếp và trọng tâm G. Giả sử rằng Hình minh họa (hinh 7). . Chứng minh rằng. song song với. ..

<span class='text_page_counter'>(22)</span> Kéo dài Ta có:. cắt. tại . Khi đó . Lại có :. là điểm chính giữa cung. Do suy ra sđ cung Từ Áp dụng định lí Ptô-lê-mê cho tứ giác nội tiếp. (không chứa. ).. ta có:. Từ Áp dụng tính chất đường phân giác trong tam giác và (5) ta có:. Vậy Mặt khác G là trọng tâm của tam giác suy ra Từ Suy ra IG là đường trung bình của tam giác. hay. song song với. .. Đây là một bài toán khá là hay ít nhất là đối với THCS và với cách làm có vẻ "ngắn gọn" này ta đã phần nào hình dung được vẻ đẹp của các định lí. Bài toán 3: Cho tam giác ABC nội tiếp đường tròn (O), CM là trung tuyến. Các tiếp tuyến tại A và B của (O) cắt nhau ở D. Chứng minh rằng:. Hình minh họa hinh 8).

<span class='text_page_counter'>(23)</span> Chứng minh: Gọi N là giao điểm của CD với (O). Xét tam giác DNB và DBC có: chung.. Tương tự ta cũng có : Mà nên từ Áp dụng định lí Ptô-lê-mê cho tứ giác nội tiếp. ta có:. Từ (3) và giả thiết Xét. và. có:. Vậy bài toán được chứng minh. Cơ sở để ta giải quyết các bài toán dạng này là tạo ra các tứ giác nội tiếp để áp dụng định lí sau đó sử dụng lí thuyết đồng dạng để tìm ra mối quan hệ giữa các đại lượng. Đây là một lối suy biến ngược trong hình học. 3, Chứng minh các đẳng thức hình học: Bài toán 1: Giả sử. là các điểm nằm trong . Chứng minh rằng:. Hình minh họa: (hinh 9). Chứng minh:. sao cho.

<span class='text_page_counter'>(24)</span> Lấy điểm K trên đường thẳng BN sao cho. Mặt khác dễ thấy rằng .. Cũng từ. , từ đó. và. suy ra:. dẫn đến. ta có:. . suy ra tứ giác nội tiếp đường tròn. Áp dụng định lí Ptô-lê-mê cho tứ giác Nhưng từ. , lúc đó. ta có:. thì :. Nên ta có đẳng thức (3) Đây là 1 trong những bài toán khá là cổ điển của IMO Shortlist. Ta vẫn có thể giải quyết bài toán theo một hướng khác nhưng dài và phức tạp hơn đó là sử dụng bổ đề: Nếu M,N là các điểm thuộc cạnh BC của. sao cho. thì . Đây là một bổ đề mà các bạn cũng nên ghi nhớ.. Bài toán 2: Cho tứ giác ABCD nội tiếp trong đường tròn (O). Chứng minh rằng:. Hình minh họa hinh 10). Chứng minh: Lấy E và F thuộc đường tròn sao cho: Khi đó: Áp dụng định lí Ptô-lê-mê cho hai tứ giác nội tiếp AECD và BCDF ta có:. Mặt khác: Do đó: Suy ra: Từ (1), (2), (3) ta có điều phải chứng minh. Bài toán 3: Cho tam giác ABC với BE, CF là các đường phân giác trong. Các tia EF, FE cắt đường tròn ngoại tiếp tam giác theo thứ tự tại M và N. Chứng minh rằng:.

<span class='text_page_counter'>(25)</span> Hình minh hoạ (hình 11). Chứng minh: Đặt Áp dụng định lí Ptô-lê-mê cho hai tứ giác nội tiếp. và. ta có:. Từ (1) và (2) ta được: Mặt khác ta lại có: Tương tự : Từ (4), (5) và tính chất đường phân giác ta có: Chứng minh tương tự ta được: Từ (3), (6), (7) ta có điều phải chứng minh. Có thể dễ dàng nhận ra nét tương đồng giữa cách giải của 3 bài toán đó là vận dụng cách vẻ hình phụ tạo ra các cặp góc bằng các cặp góc cho sẵn từ đó tìm ra các biểu diễn liên quan. Một đường lối rất hay được sử dụng trong các bài toán dạng này. 4, Chứng minh bất đẳng thức và giải toán cực trị trong hình học: Bài toán 1: (Thi HSG các vùng của Mĩ, năm 1987) Cho một tứ giác nội tiếp có các cạnh liên tiếp bằng minh rằng:. Chứng minh: Áp dụng định lí Ptô-lê-mê cho tứ giác nội tiếp thì. và các đường chéo bằng. Chứng.

<span class='text_page_counter'>(26)</span> Vậy ta cần chứng minh Bất đẳng thức này chính là một bất đẳng thức rất quen thuộc mà có lẽ ai cũng biết đó là bất đẳng thức Bunhiacopxki-BCS. Vậy bài toán được chứng minh. Một lời giải đẹp và vô cùng gọn nhẹ cho 1 bài toán tưởng chừng như là khó. Ý tưởng ở đây là đưa bất đẳng thức cần chứng minh về 1 dạng đơn giản hơn và thuần đại số hơn. Thật thú vị là bất đẳng thức đó lại là BCS. Bài toán 2: Cho lục giác lồi ABCDEF thỏa mãn điều kiện Chứng minh rằng: HÌNH MINH HỌA (hinh 12). Chứng minh: Đặt. Áp dụng định lí Ptô-lê-mê mở rộng cho tứ giác . Vì nên suy ra:. ta có:. Tương tự ta cũng có: Từ đó suy ra Bất đẳng thức đã qui về dạng chính tắc SOS : Dễ thấy:. Như vậy , đánh giá tương tự ta cũng dễ dàng thu được kết quả Vậy bất đẳng thức được chứng minh. Đẳng thức xảy ra khi và chỉ khi Tức là khi ABCDEF là một lục giác đều nội tiếp.. . .. Đây là một bài toán do zaizai phát triển từ một bài toán quen thuộc. Nó cũng xuất phát từ bài.

<span class='text_page_counter'>(27)</span> Stronger than Nesbit inequality của mình. :lol: Cơ sở khi giải bài toán này là sử dụng phương pháp SOS để làm mạnh bài toán. Với bước chuyển từ việc chứng minh 1 bất đẳng thức hình học sang bất đẳng thức đại số ta dễ dàng tìm ra 1 lời giải đẹp. Nếu chuẩn hóa bất đẳng thứ này ta cũng có kết quả rất thú vị. Bài toán 3: Cho lục giác lồi ABCDEF thỏa mãn điều kiện cạnh bằng Chứng minh rằng:. và tổng độ dài ba. Lời giải: Ta chuyển việc chứng minh bất đẳng thức trên về chứng minh bất đẳng thức sau:. Bằng cách sử dụng phương pháp hệ số bất định ta dễ dàng tìm được bất đẳng thức phụ đúng:. Tương tự với các phân thức còn lại ta có điều phải chứng minh. Khi định hướng giải bài này chắc hẳn bạn sẽ liên tưởng ngay đến SOS nhưng thật sự thì nó ko cần thiết trong bài toán này bởi chỉ làm phức hóa bài toán. Dùng phương pháp hệ số bất định giúp ta tìm ra 1 lời giải ngắn và rất đẹp. Tuy nhiên lời giải này ko dễ hiểu lắm đối với THCS. Thực ra cách làm mới bài toán này cũng cực kì đơn giản vì xuất phát điểm của dạng chuẩn là bất đẳng thức Nesbit quen thuộc vì vậy dễ dàng thay đổi giả thiết để biến đổi bài toán. Mà cách thay đổi điều kiện ở đây chính là bước chuẩn hóa trong chứng minh bất đẳng thức đại số. Nói chung là dùng để đồng bậc bất đẳng thức thuần nhất. Với tư tưởng như vậy ta hoàn toàn có thể xây dựng các kết quả mạnh hơn và thú vị hơn qua một vài phương pháp như SOS, hệ số bất định, dồn biến và chuẩn hóa. Đặc biệt sau khi chuẩn hóa ta có thể dùng 3 phương pháp còn lại để chứng minh. Bài toán 4:: Cho đường tròn thuộc cung lớn. và là một dây cung khác đường kính của đường tròn. Tìm điểm sao cho lớn nhất.. Lời giải: Gọi D là điểm chính giữa cung nhỏ BC. Đặt không đổi. Theo định lí Ptô-lê-mê ta có: Do và ko đổi nên đối xứng của qua tâm. lớn nhất khi và chỉ khi của đường tròn.. lớn nhất khi và chỉ khi. IV, Bài tập: Bài 1 CMO 1988, Trung Quốc) là một tứ giác nội tiếp với đường tròn ngoại tiếp có tâm ) và bán kính cắt lần lượt tại . Chứng minh rằng:. là điểm. . Các tia.

<span class='text_page_counter'>(28)</span> Bài 2: Cho đường tròn và dây cung khác đường kính. Tìm điểm A thuộc cung lớn của đường tròn để đạt giá trị lớn nhất. Bài 3: Cho tam giác ABC nội tiếp đường tròn . Đường tròn nằm trong (O) tiếp xúc với (O) tại T thuộc cung AC (ko chứa B). Kẻ các tiếp tuyến tới . Chứng minh rằng: Bài 4: Cho luc giác có các cạnh có độ dài nhỏ hơn 1. Chứng minh rằng trong ba đường chéo có ít nhất một đường chéo có độ dài nhỏ hơn . Bài 5: Cho hai đường tròn đồng tâm, bán kính của đường tròn này gấp đôi bán kính của đường tròn kia. là tứ giá nội tiếp đường tròn nhỏ. Các tia lần lượt cắt đường tròn lớn tại . Chứng minh rằng: chu vi tứ giác lớn hơn 2 lần chu vi tứ giác .. I.11) Định lý Pascal Định lý: Cho 6 điểm A,B,C,D,E,F cùng thuộc một đường tròn. Khi đó các giao điểm của các cặp cạnh AB và DE, BC và EF, CD và FA thẳng hàng.. Chứng minh: Gọi P,M,N lần lượt là giao điểm của AF và CD, AB và DE, BC và EF. Gọi P', M', N' lần lượt là giao điểm của BC và DE, BC và AF, DE và AF. Áp dụng định lí Menelaus cho P'M'N' với cát tuyến PCD:. Tương tự ta có: và.

<span class='text_page_counter'>(29)</span> Nhân các biểu thức trên lại kết hợp với các biểu thức phương tích sau:. Ta có : . Áp dụng định lí Menelaus đảo ta có đpcm. Các bạn có thể vào đây xem thêm: I.12) Định lý Brianchon Định lý: Cho lục giác ABCDEF ngoại tiếp (O). Chứng minh rằng ba đường chéo lớn AD, BE, CF đồng quy.. Chứng minh: Ta kí hiệu các tiếp điểm của (O) trên AB,BC,CD,DE,EF,FA lần lượt là M,N,P,Q,R,S. Xét cực và đối cực đối với (O). Gọi K,I,J lần lượt là giao điểm của các cặp đường thẳng (SM,PQ) ,(MN,QR), (NP,RS). Vì SM và PQ là đường đối cực của A và D nên AD là đường đối cực của K. Tương tự BE và FC lần lượt là đường đối cực của I và J. Dùng định lí Pascal cho lục giác nội tiếp MNPQRS ta có I,J,K thẳng hàng. Nên ta có các đường đối cực của I,J,K (lần lượt là BE,CF,AD) cùng đi qua cực của đường thẳng này (đường thẳng đi qua I,J,K) nên AD,BE,CF đồng quy (đpcm). Tương tự ngược lại có thế chứng minh định lí pascal thông qua Brianchon và cực đối cực(xem thêm cực đối cực ở mục III.5 hoặc xem I.13)Định lí Miquel.

<span class='text_page_counter'>(30)</span> Định lí: Cho tam giác ABC và ba điểm M,N,P lần lượt nằm trên BC,CA,AB. Khi đó các đường tròn ngoại tiếp các tam giác APN,BPM và CMN đồng quy. Chứng minh:. Gọi S là giao điểm của (BPM) và (CMN).Ta sẽ chứng minh S nằm trên (APN). Thật vậy:. Suy ra điều cần chứng minh.!. I.14) Công thức Carnot Định lý: Cho ABC nội tiếp (O,R). Gọi x,y,z lần lượt là khoảng cách từ O đến BC,AC,AB. Gọi r là bán kính đường tròn nội tiếp tam giác ABC. Ta có: a)Nếu nhọn thì công thức carno là . b)Nếu thì công thức carno là Chứng minh: a)Nếu. nhọn.

<span class='text_page_counter'>(31)</span> Gọi F, E, D lần lượt là trung điểm của BC,CA,AB. Như vậy ta có . Đặt . Áp đụng bất đẳng thức Ptolemy cho tứ giác nội tiếp OFBD ta có: hay Tương tự ta có. và. ta lại có Cộng bốn biểu thức trên lại ta có. b)Nếu. chứng minh tương tự.. Viết dưới dạng lượng giác, công thức Carnot chính là hệ thức ý hệ thức này đúng với mọi tam giác.. . Chú. I.15) Định lí Carnot Định lý: Cho . Gọi lần lượt là các điểm thuộc các cạnh lượt là các đường thẳng đi qua và vuông góc với khi và chỉ khi. . .. lần đồng quy.

<span class='text_page_counter'>(32)</span> Chứng minh: a)Phần thuận: Gọi ĐPCM. đồng quy tại O.. Đẳng thức này đúng nên ta có điều phải chứng minh. b) Phần đảo Gọi giao điểm của tại O. Qua O hạ đường vuông góc xuống AB tại P'. Áp dụng định lí thuận ta có đồng quy.. P trùng với P'. I.16/Định lý Brokard Định lý: Cho tứ giác lồi ABCD nội tiếp đường tròn tâm O.AD giao BC tại M,AB giao CD tại N,AC giao BD tại I.Chứng minh rằng O là trực tâm của tam giác MIN..

<span class='text_page_counter'>(33)</span> Chứng minh: Gọi H là giao thứ 2 của hai đường tròn ngoại tiếp các tam giác AID,BIC. Xét tứ giác DOHC,ta có: Từ đó suy ra tứ giác DOHC nội tiếp.Tương tự ta cũng suy ra tứ giác AOHB nội tiếp. Dễ thấy suy ra N nằm trên trục đẳng phương của hai đường tròn Ta có:. -->. thẳng hàng.. Từ đó suy ra Tương tự ta có: Suy ra O là trực tâm tam giác MIN (đpcm). I.17) Định lí Euler về khoảng cách giữa tâm 2 đường tròn nội, ngoại tiếp của tam giác. Định lý: Cho tam giác ABC nội tiếp (O;R) và ngoại tiếp (I;r). Chứng minh rằng. ..

<span class='text_page_counter'>(34)</span> Chứng minh: Kéo dài AI cắt (O) tại M. Vẽ đường kính MN của đường tròn (O). Hạ . Kéo dài OI cắt (O) tại E và F. Ta có ~ Mặt khác dễ dàng chứng minh Lại có. . nên ta có điều phải chứng minh.. I.18)Định lí Euler về khoảng cách giữa tâm hai đường tròn nội ngoại tiếp tứ giác!(Định lí Fuss) Định lí :Cho tứ giác ABCD vừa nội tiếp (O,R) vừa ngoại tiếp (I,r). Đặt d=OI. Khi đó ta có:. Chứng minh.

<span class='text_page_counter'>(35)</span> Gọi tiếp điểm của (I) trên AB,BC,CD,DA lần lượt là M,N,P,Q. BI,CI cắt (O) lần lượt ở E,F . Ta thấy: Do đó E,O,F thẳng hàng ,nên O là trung điểm của EF. Theo công thức đường trung tuyến trong tam giác IEF ta có: Từ đó suy ra:. (vì. I.19)Định lí Casey(Định lí Ptolemy mở rộng) Định lí :Cho tứ giác ABCD nội tiếp (O,R). Đặt các đường tròn với (O) tại các đỉnh A,B,C,D. Đăt. là các đường tròn tiếp xúc. là độ dài đoạn tiếp tuyến chung của hai đường tròn. .. Trong đó là độ dài tiếp tuyến chung ngoài nếu hai đường tròn cùng tiếp xúc trong hoặc cùng tiếp xúc ngoài với (O), và là độ dài đoạn tiếp xúc trong nếu trong trường hợp còn lại. Các đoạn. , ... được xác định tương tự. Khi đó ta có:. Chứng minh Ta chứng minh trường hợp cùng tiếp xúc ngoài với (O). Các trường hợp còn lại chứng minh tương tự. Lần lượt đặt tâm các đường tròn trên là A',B',C',D' và bán kính lần lượt là x,y,z,t. Đặt AB=a, BC=b, CD=c, DA=d, AC=m, BD=n. Áp dụng định lý Pythagore: Mặt khác lại có: (theo định lí hàm số cos).

<span class='text_page_counter'>(36)</span> Tương tự với Ta có. , .... (định lý Ptolemy) Ngược lại ta thấy định lý Ptolemy là một trường hợp đặc biệt của định lí Casey khi x=y=z=t=0.. I.20)Hệ thức Stewart Định lí:Cho ba điểm A,B,C thẳng hàng. Và một điểm M bất kì. Ta luôn có hệ thức. Chứng minh Qua M hạ Ta có:. .. (Đưa về trường hợp hệ thức Stewart cho 4 điểm thẳng hàng (khi M nằm trên đường thẳng chứa A,B,C)). Ta có đpcm.. I.21)Định lí Lyness. Định lí:Nếu đường tròn tâm O tiếp xúc trong với đường tròn ngoại tiếp tam giác ABC tại T và tiếp xúc với các cạnh AB,AC của tam giác lần lượt tại E và F thì tâm đường tròn nội tiếp của tam giác nằm trên EF..

<span class='text_page_counter'>(37)</span> Chứng minh: Để chứng minh định lí này ta cần chứng minh 2 bổ đề sau: Bổ đề 1:AB là dây của một đường tròn tâm (O). Đường tròn (l) tiếp xúc với dây AB tại K và tiếp xúc trong với (O) tại T. Chứng minh L là trung điểm của cung AB ko chứa T và Bổ đề 2: Điểm M là trung điểm cung BC ko chứa A của đường tròn ngoại tiếp tam giác ABC. Điểm I thuộc đoạn MA sao cho MI=MB. Chứng minh rằng I là tâm đường tròn nội tiếp tam giác ABC. Việc chứng minh 2 bổ đề này là khá đơn giản. Ta tiếp tục quay trở lại với việc chứng minh định lí Lyness. kẻ TF giao (O) tại P; BP cắt EF tại H. Theo bổ đề 1 ta có BP là phân giác của góc B. Ta có:. nt. Mà Theo bổ đề 1 ta lại có Theo bổ đề 2 ta được H là tâm đường tròn nội tiếp tam giác ABC (ĐPCM). I.22)Định lý Lyness mở rộng(Bổ đề Sawayama) Định lí:Cho tam giác ABC nội tiếp đường tròn (O).M thuộc BC (Có cách phát biểu khác là: cho tứ giác ABDC và M là giao của BC và AD; nhưng hai cách phát biểu này là tương đương). Một đường tròn (O') tiếp xúc với hai cạnh MA và MC tại E và F đồng thời tiếp xúc với cả đường tròn (O) tại K. Khi đó ta có tâm đường tròn nội tiếp của tam giác ABC nằm trên đường thẳng EF..

<span class='text_page_counter'>(38)</span> Chứng minh KF cát đường tròn (O) tại G. Áp dụng bổ đề 1 tại bài viết của chu t tung về định lý Lyness ở trên, ta có G là điểm chính giữa cung BC. Gọi I là giao của AG với EF. Ta có nội tiếp ~ ~ Lại cũng theo bổ đề 1 ta có I là tâm nội tiếp của bổ đề 2) Xem thêm các hệ quả của định lý Lyness tại báo toán tuổi thơ 2 số 42 và 43. (theo. I.23) Định lí Thébault Định lí: Cho tam giác tròn tâm. nội tiếp đường tròn. tiếp xúc với 2 đoạn. với 2 đoạn và tiếp xúc trong với thẳng hàng. Chứng minh. .. là một điểm nằm trên cạnh. và tiếp xúc trong với . Gọi. . Đường tròn tâm. là tâm nội tiếp tam giác. . Đường tiếp xúc . Ta có:.

<span class='text_page_counter'>(39)</span> Gọi lần lượt là tiếp điểm của với . Gọi là giao điểm của và . Theo định lí lyness mở rộng(đã có trong bài của trung anh), là tâm nội tiếp tam giác . Vậy ta chỉ cần chứng minh thẳng hàng. Thật vậy, gọi lần lượt là giao điểm của và ; hàng(dpcm). và. . Áp dụng định lí Thales ta có:. . Vậy ,. thẳng. I.24)Công thức Jacobi liên quan đến tâm tỉ cự,định lí Lebnitz 1)Công thức Jacobi: Nếu I là tâm tỉ cự của hệ điểm trên mặt phẳng ta đều có:. ứng với các hệ số. thì với mọi điểm M. Chứng minh:. (do I là tâm tỉ cự của hệ điểm nên ->đpcm.. ). 2)Định lí Lebnitz Đây là trường hợp đặc biệt của công thức trên khi n=3 3)Hệ quả khác: Giá trị nhỏ nhất của biểu thức (với các kí hiệu như phần trên) đạt được khi . (phần này thuần về tính toán nên chắc không cần vẽ hình phải không anh ma29?). I.25)Định lí Newton cho tứ giác ngoại tiếp Định lý Cho tứ giác ABCD ngoại tiếp đường tròn O.Khi đó trung điểm hai đường chéo AC,BD và tâm O thẳng hàng. Hình: (vẽ bằng Cabri hơi xấu):rokeyrulez:.

<span class='text_page_counter'>(40)</span> Chứng minh Gọi P,Q,R,S lần lượt là các tiếp điểm của các đoạn thẳng AB,BC,CD,DA đối với đường tròn . Đặt .Áp dụng định lý con nhím cho tứ giác ABCD ta có:. <-> <-> <-> Từ đó suy ra hai vecto. cùng phương->O,M,N thẳng hàng (đpcm). I.26)Định lí Breichneider. (định lý hàm số cos cho tứ giác). Định lý Cho tứ giác ABCD có độ dài các cạnh AB, BC, CD, DA lần lượt là a, b, c, d và độ dài hai đường chéo AC, BD là m, n. Khi đó ta có:.

<span class='text_page_counter'>(41)</span> Chứng minh Trên cạnh AB ra phía ngoài dựng tam giác ABN đồng dạng với tam giác CAD, và dựng ra phía ngoài cạnh AD tam giác ADM đồng dạng với tam giác CAB. Khi đó dễ thấy: và BDMN là hình bình hành. Đồng thời có Áp dụng đính lí hàm số cos cho tam giác NAM, ta có. I.27)Định lí con nhím: Định lí:Cho đa giác lồi cạnh giác.Thế thì :. và các vecto là các vecto có độ dài bằng các ,tương ứng vuông góc với các cạnh ấy và hướng ra phía ngoài đa.

<span class='text_page_counter'>(42)</span> Chứng minh: Không giảm tính tổng quát ,giả sử đa giác Xét phép quay vecto:. có hướng dương.. Từ đó suy ra điều cần chứng minh.. I.28)Định lí Gergone-Euler. Định lí:Xét tam giác ABC và một điểm S trong mặt phẳng.AS,BS,CS lần lượt cắt BC,CA,AB ở D,E,F.Khi đó ta có:. Chứng minh:.

<span class='text_page_counter'>(43)</span> Kí hiệu Ta thấy :. chỉ diện tích đại số của đa giác.. (1) Tương tự ta có: (2) (3) Cộng theo vế (1),(2)và (3) ta có điều cần chứng minh.. I.29)Định lí Peletier I.30)Định lí Miobiut. Định lí:Cho ngũ giác lồi Chứng minh:. I.31)Định lí Viviani Định lí: Trong tam giác đều ABC ta lấy 1 điểm S .Ta sẽ có tổng các khoảng cách từ điểm S tới ba cạnh sẽ có độ dài bằng 1 đường cao của tam giác. Chứng minh:.

<span class='text_page_counter'>(44)</span> Kẻ SD,SE,SF lần lượt vuông góc với BC,CA,AB. Đặt độ dài cạnh tam giác ABC là a,độ dài đường cao AH là b. Ta có : Giản ước hai vế cho a ta có điều cần chứng minh.. I.32)Công thức Lagrange mở rộng. Định lý: Gọi I là tâm tỉ cự của hệ điểm. ứng với các hệ số. thì với mọi điểm M:. Chứng minh: Từ hệ thức Jacobi (có thể xem ở mục I.24) thì ta chỉ cần chứng minh rằng:. Do I là tâm tỉ cự của hệ điểm nên:. <->. <->.

<span class='text_page_counter'>(45)</span> <->. <->. (đpcm). I.33) Đường thẳng Simson Định lí:Cho và điểm nằm trên đường tròn ngoại tiếp tâm lần lượt là hình chiếu vuông góc của M trên các đường thẳng cùng thuộc một đường thẳng (đây gọi là đường thẳng Simson). Chứng minh:. Dùng góc định hướng Ta có:. Vậy. thẳng hàng.. (Xem them riegelmj[1].bdf; 00045.bdf). của tam giác. Gọi thì chúng.

<span class='text_page_counter'>(46)</span> I.34) Đường thẳng Steiner Định lí:Cho. và điểm nằm trên đường tròn ngoại tiếp tâm của tam giác. Gọi lần lượt là điểm đối xứng với của D qua các đường thẳng thì chúng cùng thuộc một đường thẳng và đường thẳng này đi qua trực tâm H của tam giác ABC. Đường thẳng đó được gọi là đường thẳng steiner ứng với điểm D của tam giác ABC. Còn điểm D được gọi là điểm anti steiner.. Chứng minh: Dễ thấy nếu gọi trung điểm của đoạn Ta có. lần lượt là hình chiếu của D xuống ba cạnh của tam giác ABC thì và tương tự ta có thẳng hàng.. là. (mod ) Vậy đường thẳng steiner đi qua H. Từ đó ta có được tính chất rằng đường thẳng simson ứng với điểm D đi qua trung điểm của đoạn DH.. I.35) Điểm Anti Steiner(Định lí Collings) Định lí:Cho và đường thẳng đi qua H trực tâm của tam giác ABC . Gọi lần lượt là đường thẳng đối xứng của d qua BC,AC,AB. Các đường thẳng đó đồng quy tại một điểm nằm trên đường tròn ngoại tiếp tam giác ABC(điểm anti steiner của d). Và d được gọi là đường thẳng steiner của điểm đó (gọi là G)..

<span class='text_page_counter'>(47)</span> Chứng minh: Gọi lần lượt là hình chiếu của H qua ba cạnh \Rightarrow ba điểm này thuộc (O) ngoại tiếp tam giác ABC và lần lượt thuộc (mo d ) Vậy nếu gọi giao điểm của d_a,d_b là G thì G thuộc đường tròn ngoại tiếp tam giác ABC. Tương tự ta có đpcm Theo hình của bài đường thẳng steiner ta dễ thấy với Vậy ta có d đúng là đường thẳng steiner của G.. đối xứng với. ,. đối xứng. Ta có một tính chất khác của điểm Anti Steiner như sau: Định lí 2: Gọi P là một điểm thuộc đường thẳng d. lần lượt là điểm đối xứng với P qua các cạnh của tam giác ABC. Ta có các đường tròn qua điểm G. Chứng minh : Dễ thấy (mod. ). cùng đi.

<span class='text_page_counter'>(48)</span> Lại có theo chứng minh trên có: (mod Suy ra G thuộc. ). . Tương tự có đpcm. I.36)Định lí Napoleon Định lí:Dựng ra phía ngoài tam giác ABC các tam giác đều BMC,CNA,APB và gọi D,E,F lần lượt là tâm của ba tam giác ấy. Khi đó ta có tam giác DEF đều. Chứng minh:. Bài này có nhiều cách giải,nếu thuận lợi mình sẽ giới thiệu ,tuy nhiên ở đây mình sẽ trình bày một chứng minh ngắn gọn dựa trên phép quay vecto như sau: Từ đó có điều cần chứng minh.. Napoleon's Theorem, Two Simple Proofs On each side of a triangle, erect an equilateral triangle, lying exterior to the original triangle. Then the segments connecting the centroids of the three equilateral triangles themselves form an equilateral triangle. By Dr. Scott Brodie, M.D., Ph.D. Mount Sinai School of Medicine, NY.

<span class='text_page_counter'>(49)</span> Proof #1 ("Hammer and Tongs" trigonometry) In the following, we use standard notations: in the the vertex A and the correposnding. ABC, A denotes both. A, a is both BC and its length. In. addition, let G denote the centroid of the equilateral triangle on side AB, and I denote the centroid of the equilateral triangle on side AC, etc. Let s denote the length of segment GI, t the length of segment AG, and u the length of segment AI. (Geometer's SketchPad illustration.) Since. IAC =. GAB = 30o, we may apply the Law of Cosines to compute s2 = u2 + t2 - 2ut·cos(A + 60o).. (1). Since the centroid of a triangle lies along each median, 2/3 of the distance from the vertex to the midpoint of the opposite side, we have t = (2/3)·sqrt(3)/2 · c = c/sqrt(3) u = (2/3)·sqrt(3)/2 · b = b/sqrt(3), and (1) becomes 3·s2 = b2 + c2 - 2bc·cos(A + 60o).. (2). Expanding(*) the cosine of the sum, and recalling that cos(60o) = 1/2; sin(60o) = sqrt(3)/2, we have cos(A + 60o) = cos(A)/2 - sin(A)·sqrt(3)/2.. (3). Substituting (3) into (2) yields 3·s2 = b2 + c2 - bc·cos(A) + sqrt(3)·bc·sin(A). Now apply the Law of Cosines to. (4). ABC: a2 = b2 + c2 - 2bc·cos(A).. and recall (as in the derivation of the Law of Sines):. (5).

<span class='text_page_counter'>(50)</span> 2·Area(. (6). ABC) = bc·sin(A).. Substituting (5) and (6) into (4) gives. 3·s2 = (1/2)(a2 + b2 + c2) + 2·sqrt(3)· Area(. ABC).. (7). Since (7) is symmetrical in a, b, and c, it follows that the triangle connecting the three centroids is equilateral, QED.. (*) Michael Lambrou has suggested a different way to proceed after obtaining (2). Apply the Law of Cosines to triangles ABE and BCE to express the side BE in two different ways: b2 + c2 - 2bc·cos(A + 60o) = a2 + b2 - 2ab·cos(C + 60o) By (2), the left hand side equals 3 times GI. Similarly, the right hand side equals 3 times IH, wherefrom GI = IH. Considering triangles ACD and ABD instead, we also obtain IH = HG, which furnishes the proof. As a bonus, we get AD = BE = CF..

<span class='text_page_counter'>(51)</span> Proof #2 (an argument by symmetrization) Notations are the same as before: let ABC be the original triangle. Choose D, E, and F exterior to. ABC so that. ADB, BEC, and AFC are equilateral triangles, with centroids G, H, and I, respectively. (Geometer's SketchPad illustration.) We proceed to show that HIG = 60o. Fix I as a center of rotation, and rotate the entire figure by 120o, and superimpose the rotated copy on the original figure. Under the rotation, the. CAF maps to itself (C. maps to A, A to F, F to C, while I maps into itself.) Denote by BB, DD, EE, GG, and HH the images of points B, D, E, G, and H, respectively. Connect D to EE and G to HH. By the rigidity of the rotation,. GHI =. GG.HH.I. In particular, GH = GG.HH.. (Geometer's SketchPad illustration.) Now consider the six triangles that converge on point A. Three of them (ABD, ACF, and A.EE.BB) are equilateral. Recollect that the angles of a triangle sum to 180o, while the angles around a point sum to 360o. Since. BB.A.F is a copy of. BCA, it follows that. D.A.EE =. ABC. Finally,. D.A.EE =. ABC,. and the pentagon A.BB.EE.D.B is congruent to the pentagon BECAD. It follows that G.HH = GH. And thus G.HH = GH = GG.HH..

<span class='text_page_counter'>(52)</span> Repeating the rotation by 120o once more, and connecting the tips of the equilateral triangles as above, we obtain the figure on the right. (Geometer's SketchPad illustration.) Arguing as above, it is clear that the central hexagon is equilateral, and that the six triangles which meet at the center of rotation are congruent. Therefore, 6·. HIG = 360o, and. HIG = 60o. Since (among the. points G,H,I) the choice of the centroid I was arbitrary, we have shown that. GHI is. equiangular, hence equilateral, QED.. I.37)Định lí Morley Định lí: Trong tam giác ABC. D,E,F lần lượt là giao điểm của các đường chia ba góc trong và cùng kề các cạnh tam giác ABC. Khi đó ta có tam giác DEF đều và được gọi là tam giác Morley.. Chứng minh: Để ngắn gọn ta đặt. và tương tự với các góc kia. Như hình vẽ kẻ các.

<span class='text_page_counter'>(53)</span> đường chia trong ở B và C và lần lượt cắt tại D,I. Dễ thấy ID là phân giác của góc. . Tại D. dựng góc sao cho Di là phân giác của góc DEF và E thuộc CI và F thuộc BI DEF đều. Lấy và lần lượt là điểm dối xứng với D qua CI và BI và dễ dàng c/m được là hình thang cân với Vì định lí Morley chỉ có một trường hợp nên em xin phép chỉ sử dụng góc thường cho nó đơn giản:. Ta lại có. là hình thang cân và thì sđ. A thuộc đường tròn. trong đường tròn ngoại tiếp . Từ đó ta có đpcm. Định lý Morley có thể mở rộng các đường chia trong thành các đường chia ngoài, và có thể là giao của đường chia trong với đường chia ngoài(mỗi trường hợp này lại cho ta một tam giác Morley khác nhau và theo thống kê có 36 tam giác Morley như vậy). Sau đó bài toán còn được phát triển và tương ứng được đặt thêm nhiều định nghĩa mới như "góc lửng", "tam giác ngoại lai", "tập hợp đẳng cấu", ... Sau đây là bài toán mở rộng nhất định lý Morley: Nếu chia n (n nguyên dương, n 3) tất cả các góc của một đa giác m cạnh, thì tất cả các giao của các đường thẳng là các đỉnh phân biệt của một hệ. đa giác n cạnh đều, có. thể phân chia làm họ, mỗi họ có đa giác có tâm thẳng hàng. Cách chứng minh và các khái niệm liên quan xin xem thêm tại sách "Lãng mạn toán học" tác giả Hoàng Quý nhà xuất bản giáo dục (Ai có ebook của quyển này up lên thì tốt quá.. I.38)Định lí con bướm với đường tròn Định lí: Cho đường tròn (O) và dây cung AB. I là trung điểm của AB. Qua I vẽ hai dây cung tùy ý MN và PQ sao cho MP và NQ cắt AB tại E,F. Khi đó I là trung điểm của EF.. Chứng minh: Gọi K,T là trung điểm MP và NQ. Nên OIEK, OIFT là tứ giác nội tiếp.

<span class='text_page_counter'>(54)</span> (mod. ). (mod. ). Ta lại có trung điểm EF. (mod. ). cân tại O. I là. I.39)Định lí con bướm với cặp đường thẳng Định lí:Cho tam giác . Lấy tại , đường thẳng đó ta có là trung điểm cưa Chứng minh:. là trung điểm của cắt tại. Áp dụng định lí menelaus trong tam giác. . Qua . Gọi. kẻ các đường thẳng cắt tại. cắt . Khi. ta có các hệ thức sau:. (1) (2) từ (1) và (2) ta có:. Vậy. là trung điểm của. . (ĐPCM). I.6) Định lý Desargues Định lý: Cho tam giác ABC và tam giác A'B'C'. Khi đó AA', BB', CC' đồng quy khi và chỉ khi các giao điểm của BC và B'C', CA và C'A', AB và A'B' thẳng hàng..

<span class='text_page_counter'>(55)</span> Chứng minh: Gọi X, Y, Z là lần lượt là các giao điểm của các cặp cạnh BC và B’C’, CA và C’A’, AB và A’B’ . Phần thuận: Giả sử các đường thẳng AA’, BB’, CC’ đồng quy tại S. Ta chứng minh X, Y, Z thẳng hàng. Áp dụng định lí Menelaus cho tam giác SBC với cát tuyến XB'C' ta có: hay Tương tự, ta có: và Nhân từng vế các đẳng thức trên lại với nhau, và theo định lí Menelaus suy ra X, Y, Z thẳng hàng. Phần đảo: Giả sử các điểm X, Y, Z thẳng hàng. Ta chứng minh các đường thẳng AA’, BB’, CC’ đồng quy. Gọi S là giao điểm của AA’ và BB’. SC cắt đường thẳng AC’ tại C”. Xét 2 tam giác ABC và A’B’C” có các đường nối các đỉnh tương ứng đồng quy, do đó theo phần thuận giao điểm của các cạnh tương ứng cũng đồng quy. Ta thấy AB cắt A’B’ tại Z, AC cắt A’C” tại Y (do A’, C’, C” thẳng hàng), suy ra giao điểm X’ của BC và B’C” phải thuộc YZ. Tức là X’ là giao của YZ và BC nên X’ trùng với X. Suy ra C” trùng với C’, hay AA’, BB’, CC’ đồng quy..

<span class='text_page_counter'>(56)</span> I.40 Định Lí Blaikie. Định lí: Cho tam giác ABC và đường thẳng d sao cho d cắt BC,CA,AB lần lượt ở M,N,P. Gọi S là 1 điểm bất kì trên d. Gọi M',N',P' lần lượt là điểm đối xứng của M,N,P qua S. Khi đó AM',BN',CP' đồng quy tại một điểm P và ta gọi P là điểm Blaikie của d và S đối với tam giác ABC.. Chứng Minh : Có thể cho nằm giữa . Giả sử cắt tại . Ta chứng mình thẳng hàng . Xét tam giác với điểm . Ta cần cm :. Xét tam giác. với điểm thẳng hàng. trên cạnh :. (1) Xét tam giác. với điểm thẳng hàng. trên cạnh :. (2) Nhân vế (1),(2) và rút gọn , chú ý Chú ý là. và. ta được : nên ta có đpcm.. I.41)Định lí chùm đường thẳng đồng quy. Định lí 1: Ba đường thẳng đồng quy thì định ra trên hai đường thẳng song song những đoạn.

<span class='text_page_counter'>(57)</span> thẳng tỉ lệ. Chứng minh:. I.42)Đường tròn Apollonius Định lí:Cho hai điểm A và B cố định. Khi đó quĩ tích điểm M sao cho đường tròn cố định được gọi là đường tròn Apollonius.. là một. Chứng minh: Lấy D,E thuộc đường thẳng AB sao cho a\Phần thuận Ta có đường kính DE. b\Phần đảo. ME là phân giác của. Lấy M' thuộc đường tròn đường kính DE Lại có. ME là phân giác của. M thuộc đường tròn. . đpcm. I.43)Định lí Blanchet. Định lí:Cho tam giác ABC có AH là đường cao ứng với cạnh BC. Gọi I là một diểm tùy ý thuộc đoạn AH.các đoạn thẳng BI,CI cắt các cạnh tam giác tại E và F.Chứng minh rằng HA là phân giác của góc EHF.

<span class='text_page_counter'>(58)</span> Chứng minh: Qua I kẻ đường thẳng song song với BC cắt AB,HF,HE,AC lần lượt tại M,N,P,Q. Ta chỉ cần chứng minh IN=IP là xong !! Theo Thales:. Do đó :. Hiển nhiên Nên IN =IP ---------------> dpcm. I.44)Mở rộng của định lí Blanchet Định lí:Cho tam giác ABC, lấy lần lượt thuộc các đoạn thẳng đồng quy tại một điểm.Gọi là giao điểm của chiếu của xuống Chứng minh:. . Chứng minh rằng. là phân giác của. sao cho 3 đường và .Gọi H là hình.

<span class='text_page_counter'>(59)</span> Sử dụng Ceva và Menelaus ta chứng minh được Theo định lí về chùm điều hòa ta lại có ------>đpcm (hệ quả của chùm điều hòa). I.45) Định lí Jacobi: Định lí: Cho tam giác ABC và các điểm ,. trên mặt phẳng sao cho: .Khi đó. , đồng quy tại điểm. Jacobi N.. Chứng minh: Do. đồng quy tại. và tương tự cho. nên áp dụng định lý Ceva dạng Sin.

<span class='text_page_counter'>(60)</span> ta có:. -> Xây dựng hai đẳng thức tương tự cho. Như vậy. rồi nhân 3 đẳng thức trên với nhau ta được:. đồng quy theo định lý Ceva dạng Sin.. I.46)Định lí Kiepert Định lí:Dựng ra phía ngoài tam giác ABC các tam giác cân đồng dạng BCM,CAN,ABP(Cân ở M,N,P).Khi ấy ta có AM,BN,CP đồng quy Chứng minh. Do các tam giác BCM,CAN,ABP cân và đồng dạng nên dễ thấy: Theo định lí Jacobi ta có điều cần chứng minh.. I.47)Định lí Kariya Định lí : Cho tam giác ABC nhận (I) là đường tròn nội tiếp.Ở phía ngoài tam giác lấy các điểm M,N,P sao.

<span class='text_page_counter'>(61)</span> cho IM =IN=IP và IM,IN,IP tương ứng vuông góc BC,CA,AB.Khi đó ta có AM,BN,CP đồng quy. Chứng minh:. Từ giả thiết bài toán dễ suy ra: (c.g.c) Do đó: => Tương tự: Theo định lí Jacobi ta có điều cần chứng minh.. I.48)Cực trực giao Đây là một khái niệm mở rộng kết quả về trực tâm tam giác. Định lí: Cho tam giác ABC. d là một đường thẳng bất kì trong mặt phẳng. Gọi lần lượt là hình chiếu của A,B,C trên d. Gọi lần lượ là hình chiếu của trên BC,CA,AB. Khi đó đồng quy tại một điểm gọi là cực trực giao của đường thẳng d đối với ABC..

<span class='text_page_counter'>(62)</span> Chứng minh: Áp dụng định lí carnot ta có đpcm. Trực tâm là trường hợp khi d trùng với một trong ba cạnh của. (hiển nhiên đúng) ABC. (Xem them FG200406.bdf). I.49)Khái niệm tam giác hình chiếu ,công thức Euler về diện tích tam giác hình chiếu Định lí: Cho là đường tròn nội tiếp tam giác ABC.Cho điểm M nằm trong tam giác.Gọi là hình chiếu của M lên ba cạnh BC,AC,AB. Khi đó ta gọi là tam giác hình chiếu của điểm M đối với tam giác ABC. Ta có công thức Euler về diện tích của tam giác hình chiếu:.

<span class='text_page_counter'>(63)</span> Chứng minh:(Mashimaru) Ta có tứ giác nội tiếp đường tròn đường kính nên thì . Gọi là giao điểm của với đường tròn ngoại tiếp tam giác và. Mặt khác:. Xét tam giác. , tương tự . Ta có:. .. , theo định lý sin ta có:. Suy ra:. . Từ đó ta có:. Mặt khác, ta lại có: nên:. và. (Xem them pedal(2).bdf). I.50)Khái niệm hai điểm đẳng giác Định lí: Cho tam giác ABC. M là một điểm nằm trong tam giác. 1/Khi đó các đường thẳng đối xứng với AM,BM,CM qua tia phân giác đồng quy tại M'. M' được gọi là điểm đẳng giác của M. 2/Lần lượt đặt D,E,F và D',E',F' là chân các đường cao hạ từ M và M' xuông BC,AC,AB. a/Khi đó D,E,F,D'E',F' cùng thuộc một đường tròn tâm O. Và O là trung điểm của M và M'. b/Khi đó cũng có Và. ..

<span class='text_page_counter'>(64)</span> Chứng minh: 1/(hình 1). Tương tự áp dụng định lí Ceva thuận và đảo ta có đpcm1 2/ a/(hình 2). (mod. ) (mod. ).

<span class='text_page_counter'>(65)</span> (mod. ) (mod. ). (mod ) nội tiếp. Trung trực FF' và DD' gặp nhau tại trung điểm O của MM'(t/c đường trung bình hình thang) F,F'D,D' thuộc đường tròn tâm O. Tương tự ta có đpcm. b/(hình 3). lại có Tương tự ta có đpcm. (g.g). Trường hợp M là trọng tâm thì M' là điểm đối trung (AM,BM,CM lần lượt được gọi là các đường đối trung) có tính chất rằng:. min. I.51)Khái niệm tứ giác toàn phần . Khái niệm: Một tứ giác toàn phần là một hình được tạo nên bởi bốn đường thẳng, từng đôi một cắt nhau nhưng không có ba đường nào đồng qui. Một hình tứ giác toàn phần có 4 cạnh là 4 đường thẳng ấy, có 6 đỉnh là 6 giao điểm của chúng và 3 đường chéo là 3 đoạn đi qua đỉnh đối diện (chú ý hai đỉnh này không cùng thuộc một cạnh). Chúng ta có một kết quả cơ bản và thú vị về tứ giác này như sau: Định lí :Trong hình tứ giác toàn phần cặp đỉnh đối diện nằm trên một đường chéo và cặp giao điểm của đường chéo đó với hai đường chéo còn lại lập thành một hàng điểm điều hòa..

<span class='text_page_counter'>(66)</span> Xem hình vẽ sau:. I.52)Đường thẳng Droz-Farny Định lí: Cho hai đường thẳng bất kì vuông góc với nhau tại trực tâm của tam giác ABC. Chúng tương ứng cắt các cạnh BC, AC, AB tại X, X'; Y, Y'; Z, Z'. Khi đó ta có tương ứng là các trung điểm của. thẳng hàng.

<span class='text_page_counter'>(67)</span> Chứng minh:(dịch từ bài viết của Jean-Louis Ayme) Đặt C là đường tròn ngoại tiếp ABC. là đường tròn ngoại tiếp HXX' và là điểm đối xứng với H qua BC. Tương tự với các đường tròn khác. có tâm lần lượt là .XX' là đường kính của đường tròn nằm trên đường tròn này là giao của C và và . Áp dụng định lí Collings(trong bài viết điểm Anti-steiner) với đường thẳng XYZ đi qua H, ta có đồng quy tại N trên C. Áp dụng định lí Miquel cho tam giác XNY với các điểm cùng đi qua M. Tương tự cũng đi qua M. Như vậy cùng đi qua H và M (đồng trục). Nên tâm của chúng thẳng hàng. Các tài liệu có liên quan: (Xem them FG200426.bdf; FG200427.bdf). I.53)Đường tròn Droz-Farny Định lí: Cho điểm P bất kì và tam giác ABC. Điểm Q là điểm đẳng giác với P đối với tam giác ABC. Chân các đường vuông góc với các cạnh BC,AC,AB của P là . Lấy làm tâm vẽ đường tròn đi qua Q cắt BC tại . định nghĩa tương tự. Khi đó cùng thuộc đường tròn tâm P.. Chứng minh: O là trung điểm của PQ Ta đã biết O cách đều Mà :. Hoàn toàn tương tự ta sẽ có các điểm (điều cần chứng minh).. (công thức đường trung tuyến) cách đều P.

<span class='text_page_counter'>(68)</span> I.54)Định lí Van Aubel về tứ giác và các hình vuông dựng trên cạnh. Định lí:Về phía ngoài tứ giác ABCD ta dựng các hình vuông ABUI,BCQP,CDJW,DAFE với các tâm tương ứng là T,N,V,M.Khi đó ta có TV và MN vuông góc với nhau. Chứng minh:. Gọi O là trung điểm đoạn AC. Ta thấy:. Do vậy: Chú ý F là phép đồng dạng góc góc . (1) Như vậy : (2) Tương tự ta có: (3). với tỉ số 1 có O là điểm bất động tức là F là phép quay tâm O. Từ (1) ,(2) và (3) ta có điều cần chứng minh..

<span class='text_page_counter'>(69)</span> I.2 Mở rộng định lí Menelaus theo diện tích. Định lí:Cho tam giác ABC và 3 điểm M,N,P lần lượt nằm trên BC,CA,AB.Khi đó ta có:. Chứng minh Gọi là vector chỉ phương của Ta có:. mặt khác :. tương tự:. Ta suy ra:. I.55)Hệ thức Van Aubel Định lí:Cho tam giác ABC và các điểm D,E,F lần lượt thuộc BC,CA,AB sao cho AD,BE,CF đồng quy ở S.Khi đó ta có:. Và 2 hệ thức tương tự. Chứng minh:.

<span class='text_page_counter'>(70)</span> Cách chứng minh dưới đây rất độc đáo mình đọc trong một file của anh Darij Grinberg(xem cuối bài) Chúng ta sử dụng định lí Gergonne -Euler(mục I.28)cho điểm A với tam giác SBC. Ta có:. =>. => (Xem them GergonneEuler.bdf). I.56)Định lí Pithot Định lí: Tứ giác lồi ABCD là tứ giác ngoại tiếp khi và chỉ khi :AB+CD =BC+DA Chứng minh: +)Chiều thuận: ABCD là tứ giác ngoại tiếp thì: AB+CD=BC+DA Phần này chứng minh hoàn toàn đơn giản xin dành cho bạn đọc. +)Chiều đảo:Nếu tứ giác lồi ABCD thỏa mãn : AB+CD=BC+DA (*)thì nó là tứ giác ngoại tiếp..

<span class='text_page_counter'>(71)</span> Không giảm tổng quát giả sử : ,khi ấy từ (*) cũng suy ra: Khi ấy trên AD,DC tương ứng tồn tại hai điểm N,M sao cho:AN=AB,CM=CB. Từ (*) sẽ có: DN=DM.. .. Do đó các đường phân giác của các góc tại đỉnh A,D,C sẽ là ba đường trung trực của tam giác BMN nên chúng đồng quy tại một điểm (mà ta kí hiệu là O). Nhận thấy O cách đều 4 cạnh tứ giác nên ta có điều cần chứng minh.. I.57)Định lí Johnson Định lí:Cho ba đường tròn có cùng bán kính R với tâm lần lượt là M,N,P và cùng đi qua một điểm A.Khi ấy ba giao điểm khác A của ba đường tròn ấy cùng nằm trên một đường tròn có bán kính là R. Chứng minh:.

<span class='text_page_counter'>(72)</span> Mình gặp định lí này và không hề biết chứng minh của nó ,ở đây mình trình bày một phép chứng minh mà mình nghĩ ra như sau mà theo mình nó là một chứng minh dài. Chúng ta kí hiệu các giao điểm khác A là B,C,D như hình vẽ và gọi Q là tâm (BCD). Ta thấy : PA=PB=MA=MB nên MAPB là hình thoi =>M là điểm đối xứng của P qua AB. Chú ý bán kính của (P) và (M) là bằng nhau nên suy ra (P) và (M) đối xứng với nhau qua AB.(1) Tương tự (P) và (N) đối xứng với nhau qua AC (2). (P) là đường tròn ngoại tiếp tam giác ABC (3) Từ (1) ,(2) và (3) ta sẽ suy ra D là trực tâm tam giác ABC. Đến đây công việc còn lại đã rất đơn giản......................... I.58)Định lí Eyeball Định lí: Cho hai đường tròn(O) và (O') ngoài nhau.Hai tiếp tuyến kẻ từ O tới (O') cắt (O') tại C,D.Hai tiếp tuyến kẻ từ O' tới (O) cắt (O) tại A,B.Khi đó ta có : AB =CD. Chứng minh: Chúng ta kí hiệu các điểm A,B,C,D như hình vẽ. OM,O'N tương ứng là các tiếp tuyến của (O') và (O). AD cắt lại (O') ở P. Ta thấy: (1) (2) Cộng theo vế (1) và (2) ta có:. Do đó CP là đường kính của (O'). Vì vậy sẽ có: Hoàn toàn tương tự ta cũng có các đẳng thức góc khác nên sẽ có ABCD là hình chữ nhật => AB=CD.. I.59)Bổ đề Haruki Bổ đề:Cho AB và CD là hai dây cung không cắt nhau của cùng một đường tròn và P là một điểm bất kì trên cung AB không chứa CD của đường tròn ấy.Gọi E và F lần lượt là giao điểm của PC,PD với AB.Thế thì giá trị biểu thức sau là không đổi:.

<span class='text_page_counter'>(73)</span> Chứng minh:. (AED) cắt lại AB ở G. Ta thấy:. (Không đổi) => G cố định => BG không đổi (1). Mặt khác :. (2) Từ (1) và (2 ) suy ra dpcm. (Xem them FG200809.bdf). I.60)Bài toán Langley Bài toán:Cho. cân tại A có ,. Lời giải:. .Tính. . Trên cạnh ?. lấy điểm. sao cho.

<span class='text_page_counter'>(74)</span> Đặt Trên cạnh. lấy điểm. Mặt khác. cân tại. sao cho. .Khi đó. (do. cân tại ). Suy ra Do đó. đều(. Lại có. cân tại. ). Ta suy ra cân tại =. I.61)Định lí Paul Yiu về đường tròn bàng tiếp Định lí:Cho qua. các đường tròn bàng tiếp góc tiếp xúc với 3 cạnh lần lượt tại .Các đường thẳng qua giao nhau tại .Các đường thẳng giao nhau tại .Chứng minh rằng các bộ ba điểm thẳng hàng và các đường thẳng qua chúng đồng quy.. Chứng minh:.

<span class='text_page_counter'>(75)</span> Gọi. là giao điểm của. và. với. Áp dụng định lí Menelaus cho bộ 3 điểm tương ứng với. tương ứng với. và bộ 3 điểm. ta được:. Nhân (1) và (2) cho ta:. Mặt khác Áp dụng định lí Menelaus cho. Nhân (1) với (2) ta được:. Ta suy ra:. Vậy 3 điểm. thẳng hàng. với bộ 3 điểm. và. ta được:.

<span class='text_page_counter'>(76)</span> Các bộ 3 điểm còn lại chứng minh tương tự. Chứng minh 3 đường thẳng đồng quy Gọi là lần lượt là giao điểm của Như chứng minh ở trên ta có:. tương tự với. với. thì:. Ta suy ra:. Do đó. thẳng hàng. Mặt khác theo định lí Desagues thì các đường thẳng dpcm. đồng quy.Ta suy ra. I.62)Định lí Maxwell Định lí: Cho ABC và một điểm P, các cạnh của A'B'C' song song với các đường thẳng đi qua một đỉnh ABC và điểm P. Qua A',B',C' kẻ các đường thẳng song song với các cạnh của ABC. Khi đó ta có các đường thẳng này đồng quy tại một điểm P'.. Chứng minh: Dễ dàng c/m được các góc Ceva Sin ta có đpcm.. và. . Tương tự áp dụng định lí. Projective Proof of Maxwell's Theorem.

<span class='text_page_counter'>(77)</span> Maxwell's theorem links the sides and cevians in one triangle to the cevians and side lines in another:. Given two triangles ABC and MNP. Assume that the cevians in ΔABC parallel to the sides of ΔMNP are concurrent. Then the cevians in ΔMNP parallel to the sides of ΔABC are also concurrent. Michel Cabart came up with a generalization that shows that Maxwell's theorem is of projective nature. The main tool is the projective reformulation of Ceva's theorem.. By Michel Cabart 9 May, 2008 Recollect the projective formulation of Ceva's theorem: Given an arbitrary line L, the cevians in ΔABC concur iff (1). R = (CBA'A0)·(ACB'B0)·(BAC'C0) = -1. where (WXYZ) denotes the cross-ratio YW/YX : ZW/ZX and the points A', B', C', A0, B0, C0 are defined as in the diagram below, where L is an arbitrary straight line..

<span class='text_page_counter'>(78)</span> In addition, let A1, B1, C1 be the intersection with L of the cevians AA', BB', and CC'. Projection from A to the line L send the first quadruple in (1) onto B0, C0, A1, A0: (CBA'A0) = A(B0C0A1A0) Similarly, (ACB'B0) = B(C0A0B1B0), (BAC'C0) = C(A0B0C1C0) Thus the condition (1) can be written (cross-ratios being unchanged): (2). R = (A0B0C1C0)·(B0C0A1A0)·(C0A0B1B0) = -1. Proof of Maxwell's theorem.

<span class='text_page_counter'>(79)</span> Assume there are two triangles ABC and MNP and two triples of points on L such that A 0, B0, C0 are intersections of L with sides of ΔABC and also with cevians of ΔMNP; A 1, B1, C1 are intersections of L with sides of ΔMNP and also with cevians of ΔABC. For ΔABC, we calculate R0 = (A0B0C1C0)·(B0C0A1A0)·(C0A0B1B0). For ΔMNP, we calculate R1 = (A1B1C0C1).(B1C1A0A1).(A1B1C0C1) We have the identity R0·R1 = 1 (as can be seen by developing cross-ratios). Hence R0 = -1 iff R1 = -1, or: the cevians in ΔABC concur iff the cevians in ΔMNP concur. Choosing the infinite line as line L gives Maxwell's theorem as a particular case. The case where sides and cevians in ΔMNP are taken perpendicular (or in fact any angle) to ΔABC is straightforward via an homography (in projective terms) or a rotation (in Euclidean terms). Note: a dynamic illustration is available on a separate page..

<span class='text_page_counter'>(80)</span> Maxwell Theorem via the Center of Gravity Michel Cabart 30 April, 2008 Below we offer a proof of Maxwell's theorem that is based on the notion of barycenter.. Maxwell's. theorem states the following fact: Given ΔABC and a point G, the sides of ΔMNP are parallel to the cevians in ΔABC through G. Then the cevians in ΔMNP parallel to the sides of ΔABC are concurrent.. Below, the vector joining point A to B will be written in bold, so that, for example, AB = - BA. In ΔABC with A', B', C' on sides opposite the vertices A, B, and C, the fact that the cevians AA', BB', CC' concur in point G is equivalent to either of the two conditions: There is a triple of real numbers (a, b, c), unique up to a non-zero factor, such that (1). aGA + bGB + cGC = 0. There is a triple of real numbers (a, b, c), unique up to a non-zero factor, such that A' = Z(B, b; C, c) (2). B' = Z(A, a; C, c) C' = Z(A, a; B, b),.

<span class='text_page_counter'>(81)</span> where Z(X, x; Y, y) denotes the barycenter of two material points X and Y with masses x at X and y at Y. Let's suppose lines AA', BB' and CC' intersect. Step 1: NP, PM, MN are parallel to GA, GB, GC means there exists x, y, z such that NP = xGA, PM = yGB, MN = zGC. As NP + PM + MN = 0, xGA + yGB + zGC = 0. A comparison with (1) shows that (x, y, z) is a multiple of (a, b, c). We can assume (x, y, z) = (a, b, c). Thus NP = aGA, PM = bGB, MN = cGC. Step 2: MM', NN', PP' are parallel to BC, AC, AB meaning there is (m, n, p) such that MM' = mBC, NN' = nAC, PP' = pAB. The first equality yields. MM' = m(GC - GB) = (m/c)MN + (m/b)MP so that M' = Z(N, 1/c; P, 1/b) = Z(N, b; C, c) by multiplying by bc. Similarly, N' = Z(M, a; P, c) and P' = Z(M, a; N, b). This proves the theorem thanks to (2).. Maxwell's Theorem The applet suggests the following theorem [Prasolov, 11.48, Pedoe, 6.1, 8.3, 28.4]:. Given ΔABC and a point P, the sides of ΔA'B'C' are parallel to the cevians in ΔABC through P. Prove that the cevians in ΔA'B'C' parallel to the sides of ΔABC are concurrent. A similar statement is true if the lines are taken to be perpendicular, instead of parallel. Furthermore, the two are clearly equivalent. We'll prove the latter. Triangles A'B'C' with sides parallel to the given set of cevians are all similar. It therefore sufficient to establish the theorem for any one of those triangles. Consider the circumcenters O A, OB, and OC, of triangles PBC, PAC, PAB, respectively. The circumcircles of triangles PBC and PAC share chord PC, so that.

<span class='text_page_counter'>(82)</span> OAOB is perpendicular to PC. Similarly, OAOC PB and OBOC PA, which means that ΔOAOBOC is one of the family A'B'C'. The lines through its vertices perpendicular to the sides of ΔABC are exactly the perpendicular bisectors of the latter, which are known to meet at a point. Two triangles ABC and A'B'C' are said to be orthologic if perpendiculars from A, B, C to B'C', A'C', A'B' are concurrent. The point of concurrency is known as the orthologic center of ΔABC with respect to ΔA'B'C'. Maxwell's theorem justifies the symmetry of the definition: if the perpendiculars from the vertices of one of the triangle to the sides of the other are concurrent, then the perpendiculars from the vertices of the latter to the sides of the former are also concurrent. As an example, in any triangle, the associated medial and contact triangles are orthologic. We can use complex variables and the real product [Andreescu, 4.1] of complex numbers to easily establish the perpendicular case of Maxwell's theorem. For two complex numbers u and v define the real product as u·v = (uv* + vu*)/2, where the asterisk denotes the complex conjugate. Assuming u = u 1 + iu2 and v = v1 + iv2, u·v = u1v1 + u2v2. In other words, the real product of two complex numbers is exactly the scalar product of the 2D-vectors represented by these complex numbers. It follows that for four complex numbers u, U, v, V the lines joining them pairwise are perpendicular iff (U - u)·(V - v) = 0. Assume now the complex coordinates of the vertices of triangles ABC and A'B'C' are a, b, c and a', b', c'. The perpendicular from A to B'C' has the equation (1). (z - a)·(c' - b') = 0.. Similarly the perpendiculars from B and C are given by the equations. (2). (z - b)·(a' - c') = 0 and (z - c)·(b' - a') = 0.. Adding the three up eliminates z:.

<span class='text_page_counter'>(83)</span> (3). a·(c' - b') + b·(a' - c') + c·(b' - a') = 0,. which is the condition for the perpendiculars from A, B, C to B'C', A'C', A'B' to be concurrent. Indeed, if (3) holds then, for any z,. (4). (z - a)·(c' - b') + (z - b)·(a' - c') + (z - c)·(b' - a') = 0.. Now let's see if we can choose z the right way. First, for any z on the perpendicular from a to b'c', the first term in (4) vanishes. The choice of z as the intersection of the perpendiculars from a and b, eliminates the first two terms, which makes the third term also 0. Importantly, (3) can be regrouped into (3'). a'·(c - b) + b'·(a - c) + c'·(b - a) = 0,. in which the triangles switched the roles. (3') is the condition for the concurrency of the perpendiculars from A', B', C' onto the sides of triangle ABC. Q.E.D.. As an example, the pedal triangle of any point is orthologic to the base triangle. Also, Gergonne and medial triangles associated with the given one are orthologic.. Remark 1 The theorem bears name of James Clerk Maxwell, a famous physicist. His proof is different from anything above yet beautifully simple. He drew 4 pairwise intersecting circles centered at A, B, C and P. Taken by three, the circles define 4 radical centers. Let A' be the radical center of the circle centered at B, C, P. B', C', P' are defind cyclically. Then, say, A' and P' lie on the common chord of the circles B and C, which makes A'P' perpendicular to BC, etc.. Remark 2 There is another proof of Maxwell's theorem based on the notion of barycenter and another one of a generalization that shows surprisingly the projective nature of the theorem.. References.

<span class='text_page_counter'>(84)</span> 1. T. Andreescu, D. Andrica, Complex Numbers From A to ... Z, Birkhäuser, 2006 2. D. Pedoe, Geometry: A Comprehensive Course, Dover, 1970 3. V. V. Prasolov, Problems in Planimetry, v 1, Nauka, Moscow, 1986 (Russian). I.62)Định lí Maxwell Định lí: Cho ABC và một điểm P, các cạnh của A'B'C' song song với các đường thẳng đi qua một đỉnh ABC và điểm P. Qua A',B',C' kẻ các đường thẳng song song với các cạnh của ABC. Khi đó ta có các đường thẳng này đồng quy tại một điểm P'.. Chứng minh: Dễ dàng c/m được các góc Ceva Sin ta có đpcm.. và. . Tương tự áp dụng định lí. I.63)Định lí Brahmagupta về tứ giác nội tiếp có hai đường chéo vuông góc. Định lí:Cho tứ giác nội tiếp ABCD có AC vuông góc với BD tại S. Khi đó đoạn nối trung điểm một cạnh với S sẽ vuông góc với cạnh đối diện. Chứng minh:.

<span class='text_page_counter'>(85)</span> Ta chứng minh đại diện,chẳng hạn gọi M là trung điểm BC ta cần chứng minh MS vuông góc với AD. Thật vậy,MS cắt AD ở H. Ta có: Do đó:. ,M là trung điểm BC nên MS=MC.. Dễ =>dpcm. I.64)Định lí Schooten Định lí: Cho tam giác đều ABC nhận (O) là đường tròn ngoại tiếp.Khi đó với mọi điểm S nằm trên (O) thì một trong 3 đoạn SA,SB,SC có một đoạn có độ dài bằng tổng độ dài hai đoạn còn lại. Chứng minh:.

<span class='text_page_counter'>(86)</span> Không giảm tổng quát ,giả sử S thuộc cung BC nhỏ. Ta sẽ chứng minh SA=SB+SC.Thật vậy: Gọi I là một điểm trên đoạn SA sao cho SI=SC. Đến đây công việc của chúng ta là chứng minh AI=SB (*) mà điều này thì khá đơn giản ,chỉ cần để ý một chút: Dễ thấy tam giác SIC đều nên (1) Mà:CA=CB (2) và Từ (1),(2) và (3) ta dễ suy ra (*). (3). I.65)Định lí Bottema Định lí:Về phía ngoài tam giác ABC ta dựng hai hình vuông ABDE, ACFG .Gọi M là trung điểm DF.Thế thì Vị trí điểm M không phụ thuộc vào vị trí điểm A và tam giác MBC vuông cân tại M. Chứng minh:.

<span class='text_page_counter'>(87)</span> Chúng ta sẽ chứng minh ý hai bởi từ điều đó cũng suy ra ngay ý một. Vậy công việc của chúng ta là chứng minh tam giác MBC vuông cân tại M. Bài này có nhiều cách giải,ở đây ma 29 xin trình bày bằng pp phép biến hình Kí hiệu: Ta thấy:. là chỉ phép quay tâm. góc quay ,giả sử chiều dương là ngược chiều kim đồng hồ.. Mà F là phép quay với góc quay nên tâm phép quay này chính là M. Dựa vào cách xác định tâm của tích hai phép quay ta dễ có dpcm. I.66)Định lí Pompeiu Định lí;Cho tam giác ABC đều ,và một điểm D trên mặt phẳng tam giác.Khi đó luôn tồn tại một tam giác với độ dài các cạnh là DA,DB,DC. Chứng minh:. Chúng ta sẽ dùng bất đẳng thức Ptolemy (Xem mục I.10)để giải quyết bài này một cách cực kì nhanh gọn! Bây giờ,theo nguyên lí khởi đầu cực trị trong ba đoạn DA,DB,DC sẽ có một đoạn có độ dài lớn nhất. Không giảm tổng quát ,giả sử đó là DA. Đến đây ta chỉ cần chứng minh ($)là xong. Kí hiệu a là độ dài cạnh tam giác đều ABC.

<span class='text_page_counter'>(88)</span> Sử dụng bất đẳng thức Ptolemy ta có: Đến đây thì thấy tiền ($)rồi. I.67)Định lí Zaslavsky Định lí:Cho tam giác và điểm .Tam giác là ảnh của tam giác đối xứng tâm .Từ kẻ các đường thẳng song song với nhau cắt .Chứng minh rằng thẳng hàng.. qua phép tại. Chứng minh: Từ. kẻ đường thẳng song song với. Vì nên. cắt. tại. .. là ảnh của qua phép đối xứng tâm là ảnh của qua phép đối xứng tâm. suy ra Ta có:. Ta suy ra dpcm. I.68)Định lí Archimedes Định lí:Cho là trung điểm .Chứng minh rằng. , điểm. chuyển động tùy ý trên. .Từ. kẻ.

<span class='text_page_counter'>(89)</span> Chứng minh: Trên tia dựng điểm. sao cho. .. Ta có: đồng thời suy ra Vậy đồng thời là đường cao, đường trung tuyến tam giác nên tam giác cân tại M suy ra do đó là trung điểm cạnh hay nói cách khác (dpcm). I.69) Định lí Urquhart Định lí:Cho hai bộ ba điểm thẳng hàng .Chứng minh rằng. và , khi và chỉ khi. là giao điểm của. và ..

<span class='text_page_counter'>(90)</span> Chứng minh: Đầu tiên ta cần chứng minh bổ đề sau:Trong tam giác ABC ta có nửa chu vi và a=BC.. với p là. Ta có: mặt khác . nên suy ra. do. nên nghịch đảo hai vế ta được dpcm. Trở lại bài toán gọi các góc như trên hình vẽ ta có:. (dpcm). I.70)Định lí Mairon Walters Định lí:Cho tam giác ABC và các đường thẳng chia 3 cạnh đối diện như hình vẽ.Chứng minh rằng.

<span class='text_page_counter'>(91)</span> Chứng minh:: Trước tiên ta cần chứng minh bổ đề sau:Cho tam giác thẳng chia đoạn Giả sử. sao cho theo tỉ số. điểm .Giả sử. di động trên đường giao nhau tại. thì. .. thì ta có:. mà lại do cùng phương với nên tồn tại một số sao cho Mặt khác cách biểu diễn này là duy nhất nên ta có đồng nhất thức:. suy ra: . Quay lại bài toán ban đầu ta áp dụng bổ đề nhiều lần liên tiếp ta được(đây chỉ là kĩ năng tính toán nên mình chỉ ghi kết quả các bác thông cảm).

<span class='text_page_counter'>(92)</span> suy ra: (dpcm). I.71)Định lí Poncelet về bán kính đường tròn nội tiếp,bàng tiếp trong tam giác vuông. Định lí:Cho tam giác góc. có. lần lượt là bán kính các đường tròn nội tiếp, bàng tiếp. .Chứng minh rằng: tam giác ABC vuông tại A khi và chỉ khi. Chứng minh:: Ta có:. tam giác. vuông tại. (dpcm). I.72)Ðịnh lí Hansen Ðịnh lí:Cho tam giác 1)Tam giác vuông. .Chứng minh rằng các điều kiện sau tương đương:. 2) 3). Chứng minh: Ðầu tiên ta chứng minh một vài hệ thức phụ sau:. ..

<span class='text_page_counter'>(93)</span> 1) 2) 3) Ta có:. Trở lại bài toán ban đầu ta có:. Do. nên chia cả hai vế cho. ta được". (1) Ðặt (1) tam giác. ta suy ra. vuông.. do dó.

<span class='text_page_counter'>(94)</span> (2) tương tự thay (2). như trên với chú ý. và. ta được:. (3) do (3). tam giác. lớn hơn nên vuông.. *chú thích:. I.73)Định lí Steinbart mở rộng Định lí:.Cho tam giác nội tiếp .Các tiếp tuyến của đường tròn tại .Trên (O) lấy các điểm .Chứng minh rằng khi và chỉ khi đồng quy hoặc các giao điểm của tam giác thẳng hàng.. giao nhau tại đồng quy với 3 cạnh.

<span class='text_page_counter'>(95)</span> Chứng minh: Gọi. Ta có:. mà. nên.

<span class='text_page_counter'>(96)</span> Tương tự ta suy ra:. do đó nếu vế phải bằng nói cách khác thẳng hàng. thì biểu thức trong ngoặc ở vế trái bằng hoặc đồng quy khi và chỉ khi. và ngược lại hay đồng quy hoặc. I.74)Định lí Monge & d'Alembert I Định lí:Cho 3 đường tròn có bán kính khác nhau và không chứa nhau.Tiếp tuyến chung ngoài của mỗi đường tròn giao nhau lần lượt tại .Chứng minh rằng: thẳng hàng.. Chứng minh: Vì các đường tròn có vai trò như nhau nên không mất tính tổng quát ta giả sử: .Khi đó ta có thể chứng minh được:. Suy ra: Theo định lí Menelaus ta suy ra dpcm.

<span class='text_page_counter'>(97)</span> *Chú thích: là phép vị tự tâm. tỉ số. biến. thành. I.75)Định lí Monge & d'Alembert II Định lí:Cho 3 đường tròn có bán kính khác nhau và không chứa nhau.Tiếp tuyến chung trong của (A) và (C), (B) và (C) giao nhau lần lượt tại , tiếp tuyến chung ngoài của. và. giao nhau tại. .Chứng minh rằng:. thẳng hàng.. Chứng minh: Vì các đường tròn có vai trò như nhau nên không mất tính tổng quát ta giả sử: .Khi đó ta có thể chứng minh được:. Suy ra:. Theo định lí Menelaus ta suy ra dpcm.

<span class='text_page_counter'>(98)</span> *Chú thích: là phép vị tự tâm. tỉ số. biến. thành. I.76)Định lí Steiner về bán kính các đường tròn Định lí:Chứng minh rằng trong tam giác ta có:. Chứng minh: Ta có:. (hiển nhiên) suy ra dpcm. I.77)Định lí Bellavitis Định lí::Cho tứ giác. là tứ giác điều hoà kí hiệu .Chứng minh rằng:.

<span class='text_page_counter'>(99)</span> Chứng minh: Gọi đường tròn đường kính là đường tròn Apollonius của tam giác ứng với đỉnh . Vì là tứ giác điều hoà nên do đó thuộc đường tròn Apollonius của tam giác . Dựng đối xứng với qua phân giác . Ta suy ra:. Vậy. (dpcm). I.78)Định lí Feuer bach-Luchterhand:. II/Một số điểm và đường đặc biệt được xác định duy nhất với tam giác và tứ giác II.1) Đường thẳng Euler của tam giác. Định lí:Cho tam giác gọi minh rằng: thẳng hàng.. là trực tâm, trọng tâm, tâm đường tròn ngoại tiếp.Chứng.

<span class='text_page_counter'>(100)</span> Chứng minh: Gọi là trung điểm Ta có. do đó suy ra. .. thẳng hàng. II)Đường tròn và tâm Euler Kết quả : Trong một tam giác ,trung điểm các cạnh của tam giác ,chân các đường cao và trung điểm các đoạn thẳng nối trực tâm với các đỉnh cùng nằm trên một đường tròn gọi là đường tròn Euler của tam giác ấy. Chỉ dẫn chứng minh:. Thực ra đây chỉ là một trường hợp đặc biệt của hai điểm đẳng giác (Xem mục I.50).

<span class='text_page_counter'>(101)</span> II.3)Đường đối trung, điểm Lemoine Kết quảCho tam giác của tam giác.. thì 3 đường đối trung của tam giác đồng quy tại điểm. Chỉ dẫn chứng minh:: Đường đối trung của tam giác ứng với 1 đỉnh là đường thẳng đối xứng với trung tuyến qua phân giác tương ứng của đỉnh đó. Từ định nghĩa trên áp dụng định lí Xeva dạng sin ta có:. sin( ⃗ AB ; ⃗ AA 2) sin(⃗ BA ; ⃗ BB2 ) sin (⃗ CA ; ⃗ CC 2) sin(⃗ AA 1 ; ⃗ AC) sin (⃗ BB 1 ; ⃗ BC) sin (⃗ CC1 ; ⃗ CB) . . = . . =1 sin( ⃗ AC ; ⃗ AA 2) sin( ⃗ BC ; ⃗ BB2) sin (⃗ CB ; ⃗ CC2 ) sin(⃗ AA 1 ; ⃗ AB ) sin (⃗ BB 1 ; ⃗ BA ) sin (⃗ CC1 ; ⃗ CA) Suy ra *Chú thích: kí hiệu. đồng quy tại điểm. của tam giác. tương đương với. II.4)Điểm Gergonne,điểm Nobb, đường thẳng Gergone 1)Kết quả về điểm Gergonne:Tam giác ABC với đường tròn nội tiếp (I).Tiếp điểm của (I) trên BC,CA,AB lần lượt là D,E,F.Khi đó AD,BE,CF đồng quy tại một điểm gọi là điểm Gergonne của tam giác ABC..

<span class='text_page_counter'>(102)</span> Chỉ dẫn chứng minh: Chỉ cần dùng định lí Ceva và các kết quả đơn giản : DB=DC,EA=EC,FA=FB là ra. 2)Kết quả về điểm Nobb và đường thẳng Gergonne(Vẫn với các kí hiệu trên)Một tam giác không cân có 3 điểm Nobb tương ứng là giao điểm của các cặp đường thẳng EF và CB ,DE và AB ,DF và AC. Và 3 điểm Nobb cùng nằm trên một đường thẳng gọi là đường thẳng Gergonne của tam giác ABC. Chỉ dẫn chứng minh: Xét cực và đối cực đối với (I). Đường đối cực của A là EF đi qua M,nên đường đối cực của M đi qua A. Mặt khác dễ thấy đường đối cực của M đi qua D nên suy ra đường đối cực của M là AD. Hoàn toàn tương tự ta có: Đường đối cực của N là BE và đường đối cực của P là CF Theo trên ,do AD,BE,CF đồng quy nên sẽ có điều phải chứng minh. Bình luận: Kết quả trên có thể mở rộng như sau: Cho tam giác ABC và 3 điểm D,E,F theo thứ tự thuộc BC,CA,AB sao cho AD,BE,CF đồng quy và D,E,F khác trung điểm đoạn thẳng.Gọi M,N,P lần lượt là điểm chung của các cặp đường thẳng (EF,BC) ,(DF,CA) ,(DE,AB).Khi đó M,N,P thẳng hàng Bạn có thể chứng minh kết quả trên bằng định lí Menelaus nhưng thậm chí bài toán mở rộng này cũng chỉ là trường hợp đặc biệt của định lí Desargues mà thôi!!!! (Xem them hai file : FG200821.bdf ; jcgeg200722.bdf). II.5)Điểm Nagel Kết quả:.Cho tam giác . Các đường tròn bàng tiếp xúc với 3 cạnh tương ứng đỉnh lần lượt tại thì ta có 3 đường thẳng đồng quy tại điểm của tam giác..

<span class='text_page_counter'>(103)</span> Chỉ dẫn chứng minh: Ta có:. Suy ra dpcm *Chú thích:. II.6) Điểm Brocard Định nghĩa:Trong một tam giác ABC cho trước có hai điểm Brocard M,N được xác định sao cho: và. .. II.7)Điểm Schiffler Định nghĩa:Cho tam giác Euler của tam giác. có. là tâm đường tròn nội tiếp tam giác.Khi đó 4 đường thẳng và đồng quy tại điểm của tam giác..

<span class='text_page_counter'>(104)</span> Chỉ dẫn chứng minh: Gọi là tâm đường tròn ngoại tiếp tam giác,. là trọng tâm tam giác,. là trung điểm. là trọng tâm tam giác cắt tại cắt tại tiếp xúc với cạnh tại cắt tại , cắt tại . Rõ ràng là tâm đường tròn ngoại tiếp tam giác .Do đó là đường thằng Euler của tam giác . Áp dụng định lí Menelaus cho tam giác với cát tuyến ta có: . Suy ra: Ap dụng định lí Menelaus cho tam giác. Do. với cát tuyến. ta có:. là trọng tâm tam giác IBC nên. Do đó:. Tương tự ta thấy các đường thẳng Euler của các tam giác xác định bởi hệ thức ) Vậy các đường thẳng Euler của 4 tam giác (Xem them FG200312.bdf). cũng cắt và. đồng quy tại. tại. (được.

<span class='text_page_counter'>(105)</span> II.8)Điểm Feuerbach Kết quả:Trong một tam giác ,đường tròn Euler tiếp xúc với đường tròn nội tiếp của nó,và tiếp điểm đó được gọi là điểm Feuerbach của tam giác trên.. Chỉ dẫn chứng minh:(leductam post) Gọi là tâm đường tròn Euler, ngoại tiếp, nội tiếp của tam giác là đường kính vuông góc với là hình chiếu của lên ngoại tiếp, nội tiếp. là chân đường phân giác góc 1.Đường tròn Euler tiếp xúc trong với đường tròn nội tiếp. Ta dễ dàng chứng minh được Vì. . là trực tâm, là bán kính đường tròn.

<span class='text_page_counter'>(106)</span> suy ra Chứng minh tiếp: Vì nên mà cân tại do đó Vậy Chiếu hệ thức trên lên. (1). theo phương vuông góc với. ta được:. (2) Từ (1) và (2) ta suy ra: (3) Ta có:. (4) Từ (3) và (4) ta có: Vậy. suy ra dpcm.. Hoàn toàn tương tự ta cũng có: 2.Đường tròn Euler tiếp xúc ngoài với các đường tròn bàng tiếp Từ kẻ vuông góc với do nên Từ (2) ta có: Vậy Mặt khác:. (5) nên: (6). Từ (5) và (6) ta có: Từ kẻ. (7) . Trong tam giác vuông. ta có:. (8) Từ (7) và (8) ta có:.

<span class='text_page_counter'>(107)</span> Vậy. suy ra dpcm. (Xem them BalticFeuer; GenFeuerPDF; FG200117). II.4)Điểm Gergonne,điểm Nobb, đường thẳng Gergone 1)Kết quả về điểm Gergonne:Tam giác ABC với đường tròn nội tiếp (I).Tiếp điểm của (I) trên BC,CA,AB lần lượt là D,E,F.Khi đó AD,BE,CF đồng quy tại một điểm gọi là điểm Gergonne của tam giác ABC.. Chỉ dẫn chứng minh: Chỉ cần dùng định lí Ceva và các kết quả đơn giản : DB=DC,EA=EC,FA=FB là ra. 2)Kết quả về điểm Nobb và đường thẳng Gergonne(Vẫn với các kí hiệu trên)Một tam giác không cân có 3 điểm Nobb tương ứng là giao điểm của các cặp đường thẳng EF và CB ,DE và AB ,DF và AC. Và 3 điểm Nobb cùng nằm trên một đường thẳng gọi là đường thẳng Gergonne của tam giác ABC. Chỉ dẫn chứng minh: Xét cực và đối cực đối với (I). Đường đối cực của A là EF đi qua M,nên đường đối cực của M đi qua A. Mặt khác dễ thấy đường đối cực của M đi qua D nên suy ra đường đối cực của M là AD. Hoàn toàn tương tự ta có: Đường đối cực của N là BE và đường đối cực của P là CF Theo trên ,do AD,BE,CF đồng quy nên sẽ có điều phải chứng minh. Bình luận: Kết quả trên có thể mở rộng như sau: Cho tam giác ABC và 3 điểm D,E,F theo thứ tự thuộc BC,CA,AB sao cho AD,BE,CF đồng quy và D,E,F khác trung điểm đoạn thẳng.Gọi M,N,P lần lượt là điểm chung của các cặp đường thẳng (EF,BC) ,(DF,CA) ,(DE,AB).Khi đó M,N,P thẳng hàng.

<span class='text_page_counter'>(108)</span> Bạn có thể chứng minh kết quả trên bằng định lí Menelaus nhưng thậm chí bài toán mở rộng này cũng chỉ là trường hợp đặc biệt của định lí Desargues mà thôi!!!!. II.9)Điểm Kosnita Định nghĩa.Cho tam giác là tâm đường tròn ngoại tiếp tam giác.Gọi các đường tròn ngoại tiếp tam giác .Khi đó ba đường thẳng đồng quy tại điểm của tam giác.. Chỉ dẫn chứng minh: Gọi tương ứng là giao điểm của Ta có:. Tương tự với điểm. và. là tâm và. với. rồi sau đó nhân các tỉ lệ thức với nhau ta được dpcm. II.8)Điểm Feuerbach Kết quả:Trong một tam giác ,đường tròn Euler tiếp xúc với đường tròn nội tiếp của nó,và tiếp điểm đó được gọi là điểm Feuerbach của tam giác trên..

<span class='text_page_counter'>(109)</span> Chỉ dẫn chứng minh:(leductam post) Gọi là tâm đường tròn Euler, ngoại tiếp, nội tiếp của tam giác là đường kính vuông góc với là hình chiếu của lên ngoại tiếp, nội tiếp. là chân đường phân giác góc 1.Đường tròn Euler tiếp xúc trong với đường tròn nội tiếp. Ta dễ dàng chứng minh được Vì suy ra (1) Chứng minh tiếp: Vì nên mà cân tại do đó Vậy Chiếu hệ thức trên lên theo phương vuông góc với ta được:. Từ (1) và (2) ta suy ra: (3) Ta có:. (2). . là trực tâm, là bán kính đường tròn.

<span class='text_page_counter'>(110)</span> (4) Từ (3) và (4) ta có: Vậy suy ra dpcm. Hoàn toàn tương tự ta cũng có: 2.Đường tròn Euler tiếp xúc ngoài với các đường tròn bàng tiếp Từ {I}_{a} kẻ {I}_{a}{X}_{a} vuông góc với BC do {I}_{a}S = SI nên {X}_{a}M = MN Từ (2) ta có:. Vậy Mặt khác:. (5) nên: (6). Từ (5) và (6) ta có: Từ kẻ. (7) . Trong tam giác vuông. ta có:. (8) Từ (7) và (8) ta có:. Vậy. suy ra dpcm. Bổ sung một chứng minh khác bằng phép nghịch đảo ạ: Xét. có:. + Đường tròn nội tiếp + Đường tròn. bàng tiếp trong góc. + Đường tròn Euler Kẻ tiếp tuyến chung và Ta có:. ). Gọi. tiếp xúc với. theo thứ tự tại tiếp xúc với. tại. qua trung điểm 3 cạnh của. và. .. là. tiếp xúc với chúng lần lượt tại. là tâm vị tự trong của. và. .. . . . (chú ý là.

<span class='text_page_counter'>(111)</span> Nếu. thì hiển nhiên. đã tiếp xúc với. Khi đó,. và. nên ta chỉ quan tâm đến trường hợp. nên theo hệ thức Newton,. Đặt. , lại có. .. .. nên suy ra. . Cuối cùng, xét phép nghịch đảo cực. , phương tích . Nhưng. và. tiếp xúc với. còn bản thân 2 đường tròn này bất biến qua phép nghịch đảo đang xét nên ta cũng có. cũng tiếp xúc với. và. Lập luận tương tự cho thấy. . tiếp xúc với. .. Kết thúc chứng minh! Bổ Đề: 1, tam giác. là phân giác. Điểm. 2, tứ giác nội tiếp được, khi đó và đồng quy.. nằm trong mặt phẳng tam giác thì phân giác góc. II.10)Điểm Musselman,định lí Paul Yiu về điểm Musselman Kết quả: Cho tam giác. . Các điểm. lần lượt là điểm đối xứng với. qua. các cạnh đối diện và là tâm đường tròn ngoại tiếp. Khi đó và cùng đi qua 1 điểm là ảnh của điểm qua phép nghịch đảo đường tròn ngoại tiếp tam giác . Chỉ dẫn chứng minh:.

<span class='text_page_counter'>(112)</span> Gọi và lần lượt là ảnh của điểm giác . Khi đó ta có:. và. qua phép nghịch đảo đường tròn ngoại tiếp tam. Suy ra Bây giờ ta cần chứng minh 3 điểm. ,. và. thẳng hàng.. Ta có bộ tâm tỉ cự của điểm và tâm đường tròn chín điểm do đó điểm và tâm đường tròn chín điểm là 2 điểm đẳng giác của tam giác Mặt khác - tâm đường tròn ngoại tiếp và - trực tâm cũng là 2 điểm đẳng giác của tam giác nên suy ra Ta cũng dễ dàng chứng minh được Vậy. hay nói cách khác 4 điểm. và. cùng nằm trên 1 đường tròn. Tương tự với các đường tròn còn lại ta suy ra dpcm Điểm được gọi là điểm của tam giác Định lí Paul Yiu về điểm Musselman: Với giả thiết như trên thì 3 đường tròn và Chỉ dẫn chứng minh:. cũng đi qua điểm. ..

<span class='text_page_counter'>(113)</span> Về định lí này em xin trích dẫn trực tiếp lời giải của Darij Grinberg: Theo chứng minh trên thì điểm. nằm trên. Ta có:. Mặt khác: Vậy hay và cùng nằm trên đường tròn. Tương tự với các đường tròn khác ta suy ra dpcm. và. nên ta được:.

<span class='text_page_counter'>(114)</span> II.11)Khái niệm vòng cực của tam giác. Khái niệm::Cho tam giác tù . Chân các đường cao đối diên các đỉnh là Khi đó vòng cực của tam giác là đường tròn có tâm là và bán kính xác định với Trong đó: - bán kính đường tròn ngoại tiếp - độ dài 3 cạnh tam giác - 3 góc tam giác. và. .. Ngoài ra ta có 1 tính chất của vòng cực là: cho 3 điểm bất kì chuyển động trên các đường cạnh của tam giác ABC dựng các đường tròn có đường kính là đoạn thẳng nối 1 đỉnh với điểm chuyển động trên cạnh đối diện thì khi đó vòng cực của tam giác trực giao với tất cả các đường tròn đó.. II.12)Điểm Gibert Kết quả:: Cho tam giác và điểm - ảnh của điểm qua phép nghịch đảo đường tròn ngoại tiếp tam giác . Gọi là điềm đối xứng với điểm qua các đường cạnh của tam giác. Khi đó các đường thẳng và đồng quy tại điểm nằm trên đường tròn ngoại tiếp tam giác Chỉ dẫn chứng minh:.

<span class='text_page_counter'>(115)</span> Gọi giao điểm của. và. là. thì ta có:. ∠ M 1 G j B=∠ M 1 AB +∠ M 2 BA =∠MA 1 B +∠MB1 A=π − ∠ MC1 B+ π − ∠ MC1 A =∠ AC1 B =∠C. Vậy nằm trên đường tròn ngoại tiếp tam giác ABC. Chứng minh tương tự với và ta suy ra dpcm. II13/Trục Lemoine Định lý: Cho tam giác ABC nội tiếp đường tròn .Tiếp tuyến tại A của đường tròn cắt đường thẳng BC tại X.Định nghĩa tương tự cho Y,Z.Chứng minh rằng X,Y,Z thẳng hàng và đường thẳng chứa X,Y,Z được gọi là trục Lemoine của tam giác ABC.

<span class='text_page_counter'>(116)</span> Chứng minh: Không mất tính tổng quát giả sử hai vecto đồng dạng với nhau suy ra:. Nhân hai đẳng thức trên với nhau suy ra: Do X nằm ngoài tam giác nên suy ra:. Tương tự ta có hai đẳng thức sau:. Nhân ba đẳng thức với nhau ta có:. và. cùng hướng.Do hai tam giác ABX và CXA.

<span class='text_page_counter'>(117)</span> Theo định lý Menelaus thì ba điểm X,Y,Z đồng quy. Chứng minh (2): Ta sẽ chứng minh đường đối cực của X,Y,Z đối với ABC. Edit later.... lần lượt là ba đường đối trung của tam giác. II.14)Tâm Morley Kết quả::Tâm thứ nhất được định nghĩa là tâm đường tròn ngoại tiếp của tam giác Morley thứ nhất. Tâm thứ hai được định nghĩa là tâm phối cảnh của tam giác với tam giác Morley thứ nhất. Chỉ dẫn chứng minh:. Áp dụng định lí tri Xeva cho 3 đường thẳng đồng quy là. và. ta có:. mà Suy ra. 2 1 ABC sin BCA sin BA A 1 3 3 = . sinCAA 1 1 2 sin CBA sin ACB 3 3 sin. Tương tự với các đường còn lại ta được dpcm. II.15) Tâm Spieker và đường thẳng Nagel Kết quả:Cho tam giác . Gọi đường tròn nội tiếp tam giác. là tâm. lần lượt là trung điểm của và . Tâm của tam giác ABC.Khi đó 4 điểm tâm.

<span class='text_page_counter'>(118)</span> đường tròn nội tiếp, trọng tâm, điểm của tam giác Chỉ dẫn chứng minh:. Ta có phép vị tự tâm. Xét điểm. tỉ số. và tâm. biến tam giác. cùng nằm trên đường thẳng. thành tam giác. nên suy ra. (1) có bộ số tâm tỉ cự:. (2) Từ (1) và (2) ta suy ra điểm và cùng nằm trên 1 đường thẳng. Ngoài ra ta còn 1 vài tính chất của điểm Spieker như nó là tâm đẳng phương của 3 đường tròn bàng tiếp tam giác ABC, tâm Spieker, điểm Brocard thứ 3 và điểm đẳng cự với tâm đường tròn nội tiếp thẳng hàng.... II.16)Hai điểm Fermat Kết quả:Cho tam giác . Dựng ra phía ngoài(vào trong) các tam giác đều . Khi đó tâm phối cảnh của tam giác và tam giác được gọi là điểm thứ nhất(thứ hai) hay người ta còn gọi là điểm dương(âm). Chỉ dẫn chứng minh:. và.

<span class='text_page_counter'>(119)</span> Gọi giao điểm của. Tương tự với. và. và. lần lượt với. và. là. ta có:. ta được:. AP ; ⃗ AC) sin(⃗ BM ; ⃗ BA ) sin (⃗ CN ; ⃗ CB) ZA XB YC sin( ⃗ . . = . . ZB XC YA sin( ⃗ BP ; ⃗ BC) sin(⃗ CM ; ⃗ CA ) sin (⃗ AN ; ⃗ AB) Mặt khác:. (⃗ AP; ⃗ AC)=( ⃗ AP; ⃗ AB)+(⃗ AB ; ⃗ AC)=( ⃗ AC ; ⃗ AN )+(⃗ AB ; ⃗ AC)=(⃗ AB ; ⃗ AN) mod 2 π Tương tự ta suy ra dpcm Ngoài ra:.

<span class='text_page_counter'>(120)</span> Trong đó: là lần diện tích tam giác Xem them. The Fermat Point and Generalizations P. Fermat (1601-1665) challenged Evangelista Torricelli (*) (1608-1647), the inventor of barometer with the following question Find the point such that the sum of its distances from the vertices of a triangle is a minimum. Torricelli presented several solutions. In one he observed that the circumcircles of the equilateral triangles constructed externally on the sides of a given triangle meet in a point. Many more have been found since. I'll present several solutions and two quite surprising generalizations.. Solution 1 (Published by Joseph Ehrenfried Hofmann (1900--1973) in 1929. It was independently discovered by Tibor Galai and others [Honsberger, p. 26].) In ΔABC, select a point P and connect it with vertices A, B, and C. Rotate ΔABP 60 o around B into position C'BP'. By construction, ΔBPP' is equilateral, PB = P'B, and PA = C'P'. We thus have PA + PB + PC = C'P' + P'P + PC. As the image of A under the rotation, position of C' does not depend on P. Also, PA + PB + PC. CC'. because the broken line CPP'C' is no shorter than the straight line CC'. Therefore, PA + PB + PC reaches its minimum iff P lies on CC'. For this P, that. BPC' = 60o. Had we rotated ΔABP around A, we would have found. APC' = 60o.. The result is clearly related to Napoleon's theorem. On the sides of ΔABC construct equilateral triangles ABC', ACB', and BCA'. We know that the Fermat point P that minimizes the sum PA + PB + PC lies on CC'. By the same token, it lies on AA' and BB'. Therefore, it lies on their intersection. As far as Napoleon's theorem goes, the three lines AA', BB', and CC' are concurrent. (This was already known to Thomas Simpson (1710--.

<span class='text_page_counter'>(121)</span> 1761).) Not only that but they cross at angles equal 120o. Furthermore, AA' = BB' = CC' since each of them equals PA + PB + PC for the Fermat point P. So the Fermat point is unique and lies at the intersection of three straight lines that connect vertices of the triangle with opposite vertices of Napoleon's triangles. The construction fails if one of the internal angles of ΔABC is 120o or more. In this case, the vertex corresponding to the largest angle of the triangle solves Fermat's problem. (There is a dynamic illustration pertinent to the above proof.). Solution 2. This solution only works under the assumption that the Fermat point exists and is unique. Let for a given ΔABC, P be that point. Let's move, say, A a little into position A 1 on the ray PA. Will P move along into a different position? Assuming it will and denoting its new position as P 1, we arrive at contradiction. Indeed, for ΔABC, PA + PB + PC < P1A + P1B + P1C Also, by our assumption for ΔA1BC, P1A1 + P1B + P1C < PA1 + PB + PC Summing up and cancelling, we get PA + P1A1 < PA1 + P1A Recollect that we selected A1 on the ray PA so that PA1 + A1A = PA. Which leads to.

<span class='text_page_counter'>(122)</span> AA1 + P1A1 < P1A which is of course absurd. Therefore, when A slides along the ray PA, point P does not change. Slide then vertices A and B so as to make ΔABC equilateral. This is always possible. For example, start with sliding A so as to make AB = BC. The triangle ABC becomes isosceles. It is obvious that, for an isosceles triangle, the Fermat point lies on the axis of symmetry of the triangle. For this reason, the triangle remains isosceles wherever B is located on that axis. Slide it so as to make all three sides of ΔABC equal. For an equilateral triangle, its centroid that also serves as the incenter and the circumcenter, serves as the Fermat point as well.. Solution 3 ([Honsberger, p. 26] ascribes this solution to Torricelli himself and mentions that it was rediscovered almost 300 years later by F. Riesz. In a private communication, Douglas Rogers mentioned that the solution has been also proposed by J. Steiner. Thus the same object that is most frequently referred to as Fermat's point, Torricelli's point and Fermat-Torricelli point, is sometimes also named after J. Steiner, as Steiner's point, especially in the framework of Steiner's networks. [Tikhomirov, p. 31] and [Courant and Robbins, p. 354] even refer to Fermat's problem as Steiner's. [Johnson, p. 221] attributes the present solution to Steiner.) The following solution depends on explicit knowledge of the fact that, for Fermat's point, =. AFB =. AFC = 120o. Without proving that such a point exists, it shows that, if it does, it solves Fermat's. problem.. BFC.

<span class='text_page_counter'>(123)</span> At the vertices of ΔABC draw perpendiculars to lines AF, BF, and CF. The new lines form an equilateral triangle RST. Let F1 be a point different from F. Drop perpendiculars F 1A1, F1B1 and F1C1 to the sides of ΔRST. It's a nice property (known as Viviani's theorem) of equilateral triangles that in ΔRST, FA + FB + FC = F1A1 + F1B1 + F1C1. (The sum of the distances from a point inside an equilateral triangle to the sides of the triangle does not depend on the point. ) On the other hand, obviously, F1A1 + F1B1 + F1C1 < F1A + F1B + F1C All that remains is to combine the two. (Douglas Rogers made a delightful observation. ΔRST is the largest equilateral triangle circumscribing ΔABC. According to Viviani's theorem, FA, FB, FC add up to the altitude of ΔRST. For any other equlateral ΔLMN curcumscribing ΔABC, at least one of FA, FB, or FC is not perpendicular to the side and hence is longer than such perpenduclar. It follows that the sum FA + FB + FC is bound to be larger than the alititude of ΔLMN. This implies that the side of ΔLMN is smaller than that of ΔRST. In passing, ΔRST is known as the antipedal triangle of F with respect two ΔABC.).

<span class='text_page_counter'>(124)</span> Solution 4 (The proof by Lou Talman is found on one of the discussions at the mathforum. Still very simple, it uses a little bit of analytic geometry and calculus. This is probably very close to one of Torricelli's original proofs.) One can see that the Fermat point does minimize FA + FB + FC as follows: Let z be the sum FA + FB, where F is chosen to minimize FA + FB + FC, and consider the locus of points P that satisfy PA + PB = z. This locus is an ellipse with foci at A and B. Because F minimizes z + FC, the line determined by C and F must be normal to the tangent to this ellipse at F. By the reflection property of the ellipse, the angles AFC and BFC must be equal. A similar argument shows that the angles AFC (say) and AFB must also be equal. From this it follows that the three angles are all 120 degrees.. Solution 5 (The argument is similar to that in Solution 4, but from a different perspective. [F. G.-M., p. 442] credits Lhuilier (1811) with the proof.) Assume FC is constant. Then point F that minimizes FA + FB + FC, minimizes also FA + FB and lies on the circle with center C and radius FC. The minimum is achieved for a point F on the circle for which the angles AFC and BFC are equal..

<span class='text_page_counter'>(125)</span> For more detail, draw a tangent to the circle at F and choose any point G on the tangent other than F. Let H be the intersection of HC with the circle. Then, assuming angles AFC and BFC equal, FA + FB < GA + GB < HA + HB (The latter inequality warrants extra attention. It follows from the fact the various ellipses with foci at A and B do not intersect.) From here, FA + FB + FC < HA + HB + FC = HA + HB + HC. The argument can be repeated assuming either FA or FB constant. As a result, at Fermat's point F all three angles AFC, BFC and AFB are found to be equal.. Solution 6 Fermat's point can be located with the help of Euler's generalization of Ptolemy's Theorem, see [Pedoe, pp. 93-94]..

<span class='text_page_counter'>(126)</span> Any triangle has at least two acute angles. Given ΔABC, let angles at B and C be acute. Form an equlateral triangle BCD, with D and A on opposite sides of BC. Let (O) be the circumcircle of ΔBCD. For a point Q, by Ptolemy's inequality BQ·CD + CQ·BD ≥ DQ·BC, with equality only if Q lies on (O) and such that the quadrilateral BQCD is convex. Note that, by construction, BC = CD = BD which reduces the above to BQ + CQ ≥ DQ. Therefore AQ + BQ + CQ ≥ AQ + DQ ≥ AD. Unless, Q lies on AD, AQ + BQ + CQ > AD. Let P be the intersection of AD with (O) other than D. BPCD is an inscribed convex quadrilateral and P lies on AD. So in this (and only in this) case AP + BP + CP = AD. For any other selection of Q, AQ + BQ + CQ > AP + BP + CP..

<span class='text_page_counter'>(127)</span> Angle BPC is supplementary to. BDC = 60° so that. BPC = 120°. Further, D is in the middle of the arc. BDC so that angles BPD and CPD are 60° making angles APC and APB both equal 120°. If the triangle is acute the same construction applies to the other two sides which brings up the framework of Napoleon's theorem. The three circles intersect at Fermat's point and three lines joining the vertices of ΔABC with the opposite vertices of the Napoleon triangles concur at the point P. The latter fact can be used for a more direct proof.. Solution 7 (This was a part of a solution by Grimbal to a problem posted at the wu::forum.). Construct Napoleon's equilateral triangles ABC', AB'C', A'BC externally on the sides of ΔABC. Let P be the intersection of AA' and BB'. AA' is CC' rotated clockwise 60° about B, and BB' is CC' rotated counterclockwise 60° about A, it follows that angle APB' is 60°. Let X be the point on BB' making triangle APX equilateral. Now, if BB' is rotated clockwise 60° about A, then X goes to P, B' to C, and B to C'. Hence P, C, and C' are collinear; and so AA', BB', and CC' are concurrent, and any two of these lines make an angle of 120°..

<span class='text_page_counter'>(128)</span> Searching for the Fermat point we discovered a nice property of Napoleon's triangles. I found quite surprising generalizations of those properties in a column by David Gale in The Mathematical Intelligencer which was kindly pointed out to me by Professor McWorter. Two lines passing through a vertex of a triangle are called isogonal with respect to that vertex if they form equal angles with its (internal) angle bisector. Following is the first generalization.. Theorem 1 As in the diagram, assume lines AB' and AC' are isogonal as are pairs CB', CA' and BA', BC'. Then three lines AA', BB', and CC' are concurrent, i.e., meet at a common point.. This applet requires Sun's Java VM 2 which your browser may perceive as a popup. Which it is not. If you want to see the applet work, visit Sun's website at download and install Java VM and enjoy the applet.. Buy this applet What if applet does not run? Napoleon's theorem is obtained when all three angles involved are equal to 30 o. The common point is then known as the First Napoleon point. (The Second Napoleon Point is obtained when the equlateral triangles are formed internally to the given triangle. Similarly, there are two Fermat's points, which are also known as the first and second isogonic centers.) If the base angles are just equal between themselves, the theorem bears the name of Ludwig Kiepert who replaced equilateral triangles with isosceles ones. An interesting specification of the theorem is obtained when the three angles add up to 180 o. But the theorem also admits further generalization..

<span class='text_page_counter'>(129)</span> In Theorem 1, pairs of (isogonal) lines were related to angle bisectors. As is well known from elementary geometry, angle bisectors meet at a single point (incidentally, the incenter of the triangle.) Two lines AB' and AC' through a vertex A of a triangle are said to be isotomic if they intersect the opposite side BC in points equidistant from its midpoint Ma. Theorem 1 remains valid for isotomic lines as well. And, in a certain sense, for any three concurrent lines. A second generalization that I am about to formulate and then prove belongs to the realm of Projective Geometry.. Theorem 2 Let p, q, r be concurrent lines through the vertices A, B, and C, respectively, of ΔABC. Let P A be the pencil of lines at A and let TA be the (unique) projective mapping on PA which 1. interchanges lines AB and AC, 2. leaves p fixed. Define PA, PB, and TA, TB similarly. For any line a in PA, let a' = TA(a). Similarly, for b in PB and c in PC, let b' = TB(b) and c' = TC(c). Let C' = a'. b, B' = c'. a, and A' = b'. c.. Then AA', BB', and CC' are concurrent.. Note that (an apparently more general) Theorem 2 follows from Theorem 1 with a projective mapping that leaves vertices A,B,C fixed but takes the incenter into any point P (the intersection of lines p, q, r.) To prove Theorem 2, perform the projective mapping that carries C to the origin, A to the point at infinity at y-axis, B to the point at infinity at x-axis, and P to the point (1, 1). Lines p, q, and r are carried onto the lines x = 1, y = 1, and x = y, respectively. We thus have PA is the set of all vertical lines,.

<span class='text_page_counter'>(130)</span> PB is the set of all horizontal lines, PC is the set of all lines through the origin. Transformation TA preserves the vertical direction and, therefore, is in reality a 1-dimensional projective transformation. So it's a Möbius transformation which, in general, has the form f(x) = (ax + b)/(cx + d). Transformations that interchange x = 0 with the point at infinity are given by f(x) = a + b/x (c = 1 and d = 0.) Among those, there is a single one that leaves x = 1 fixed: f(x) = 1/x. So TA maps x = a onto x = 1/a. To simplify the notations, we will denote the vertical line with x = a by a, the horizontal line y = b by b, and the line y = cx through the origin, by c. Then with similar definitions for T B and TC we have a' = TA(a) = 1/a, b' = TB(b) = 1/b, and c' = TC(c) Further. C' = a'. b = (1/a, b),. B' = c'. a = (a, a/c), and. A' = b'. c = (1/(bc), 1/b).. Thus AA' has equation x = 1/(bc), BB' has equation y = a/c, and CC' has equation y = (ab)x. Then, AA'. BB'. = (1/(bc), a/c) which lies on the line CC'.. II.17)Điểm Parry reflection. Kết quả:: Cho tam giác. . Kẻ qua. các đường thẳng. song song với nhau và. song song với đường thẳng Euler của tam giác. Gọi lần lượt là các đường đối xứng với qua và . Khi đó các đường này đồng quy tại điểm Parry reflection của tam giác ABC. Chỉ dẫn chứng minh:.

<span class='text_page_counter'>(131)</span> Gọi. là trực tâm tam giác , là ảnh của qua phép đối xứng trục và . Phép vị tự tâm tỉ số biến tam giác thành tam giác khi đó 2 đường thẳng của 2 tam giác này trùng nhau. Gọi là giao điểm của với và Ta có: HA 2= AA 2 − AH=2(AX − OP)=2 (AH+ HM −OP)=2(HM+ MN)=2 HN Vậy cũng là ảnh của qua phép đối xứng trục Dựa vào tính chất đồng dạng ta suy ra được đường thẳng qua trục. đối xứng với đường thẳng. Tương tự với các đường thẳng còn lại ta suy ra 3 đường thẳng (Xem them File FG200806.bdf). đồng quy theo định lí.

<span class='text_page_counter'>(132)</span> II.18)Đường tròn Taylor ,tâm Taylor Kết quả:.Cho tam giác các đường cao . Từ kẻ các đường vuông góc với các cạnh khi đó chân các đường vuông góc này nằm trên cùng 1 đường tròn gọi là đường tròn của tam giác Chỉ dẫn chứng minh:. Ta có: Vậy 4 điểm cùng nằm trên 1 đường tròn. Tương tự với 2 bộ 4 điểm và . Ta được: Vậy 4 điểm. cũng cùng nằm trên 1 đường tròn hay ta suy ra được dpcm. II.19)Điểm Bevan Kết quả:. Cho tam giác tròn ngoại tiếp tam giác. ,. là tâm các đường tròn bàng tiếp. Khi đó tâm đường được gọi là điểm của tam giác ..

<span class='text_page_counter'>(133)</span> Sau đây ta sẽ đến với 1 vài tính chất cua điểm : *Ta thấy rằng là tâm đường tròn nội tiếp tam giác chính là trực tâm của tam giác khi đó đường tròn ngoại tiếp tam giác ABC là đường tròn chín điểm của tam giác suy ra là trung điểm của *Tâm là trung điểm của đoạn nối trực tâm với điểm . Đầu tiên ta chứng minh 3 điểm này thẳng hàng. Xét tam giác ta có:. BI HO S p G 2 3 1 . . = . . =1 BO HG S p I 1 2 3. Vậy 3 điểm này thẳng hàng. Xét tam giác ta có 3 điểm. thẳng hàng suy ra:. Vậy ta có dpcm *Điểm là trung điểm của đoạn nối điểm và điểm . Điểm là điểm đối xứng của trực tâm qua tâm đường tròn ngoại tiếp. Khi đó tương tự như trên ta cũng có được dpcm.

<span class='text_page_counter'>(134)</span> II.20)Điểm Vecten Kết quả:. Cho tam giác . Dựng ra phía ngoài(hay vào trong) các hình vuông và . Khi đó đường nối 1 đỉnh của tam giác với tâm hình vuông dựng trên cạnh đối diện đồng quy tại điểm Vecten của tam giác . Chỉ dẫn chứng minh:. Theo định lí. thì điều này hiển nhiên và cũng dễ dàng suy ra được có 2 điểm.

<span class='text_page_counter'>(135)</span> trong và ngoài(hay âm và dương). Ngoài ra ta còn có 1 tính chất khá thú vị về điểm thẳng hàng.. : Tâm đường tròn chín điểm và 2 điểm. II.21)Điểm Mittenpunkt Kết quả: Cho tam giác , là tâm các đường tròn bàng tiếp, trung điểm các cạnh . Khi đó các đường thẳng điểm của tam giác . Chỉ dẫn chứng minh:. lần lượt là đồng quy tại. sin I c cos I a sin I b cos I a ⃗ ⃗ I a I c+ Ia Ib sin I b sin I c sin I b . cos I a sin I b . cos I a ⃗ ⃗ MC+ MB=0 Chiếu hệ thức theo phương lên trục ta được: sin I c sin I c I a A 1=⃗ I a B+ ⃗ I a C= Ta có: 2⃗. Tương tự với các đường còn lại ta suy ra dpcm.

<span class='text_page_counter'>(136)</span> B 2 C ¿ :(cos ¿2 ) 2 2 A 2 cos ¿ :¿ 2 ¿ 2 sin I b ¿ :(sin I c ¿ 2)=¿ sin I a ¿2 :¿ ¿ M =¿ cos. Ta có:. Ngoài ra ta còn có 1 vài tính chất bên lề khá thú vị: Với giả thiết như trên gọi là điểm tiếp xúc của đường tròn nội tiếp tam giác với 3 cạnh thì đồng quy. Khi đó điểm này và điểm là 2 điểm đẳng giác. *Điểm , tâm và trực tâm thẳng hàng *Điểm , tâm đường tròn nội tiêp và điểm thẳng hàng. *Điểm , trọng tâm va điểm thẳng hàng với. II.22)Điểm Napoleon. Kết quả:.Cho tam giác , dựng ra phía ngoài( hay vào trong) các tam giác đều và . Khi đó đường nối 1 đỉnh của tam giác với trọng tâm tam giác đều dựng trên cạnh đối diện đồng quy tại điểm Napoleon của tam giác . Chỉ dẫn chứng minh:. Áp dụng định lí ngoài. ta suy ra dpcm cũng dễ dàng suy ra có 2 điểm. là trong và.

<span class='text_page_counter'>(137)</span> II.23)Đường tròn Adam Kết quả:Cho tam giác ABC với điểm Gergonne G.Đường thẳng qua G song song với EF cắt AB,AC ở S,P.Đường thẳng qua G song song với DE cắt AC,BC ở Q,M.Đường thẳng qua G song song với DF cắt BA,BC ở R,N.Khi đó các điểm M,N,P,Q,R,S cùng thuộc một đường tròn gọi là đường tròn Adam của tam giác ABC.. Chỉ dẫn chứng minh:. Gọi (I) là đường tròn nội tiếp tam giác ABC,tiếp điểm của (I) trên BC,CA,AB lần lượt là D,E,F.Đường thẳng qua A và G song song với BC tương ứng cắt DE,DF ở (H,K),(V,T). Ta thấy: . Do đó AH=AK nên GT=GV. Bây giờ để ý rằng GTDN và GVDM là hai hình bình hành nên ta cũng có DM=DN..

<span class='text_page_counter'>(138)</span> Kết hợp với ID vuông góc với BC ta thu được IM=IN. Tương tự IP=IQ,IR=IS Mặt khác dễ thấy :IM=IQ=IR. Từ các khẳng định trên ta dễ nhận được điều cần chứng minh.. II.24)Tam giác Fuhrmann ,đường tròn Fuhrmann Kết quả:Cho tam giác nội tiếp đường tròn tâm . Gọi là trung điểm các cung BC, CA, AB. Lấy các điểm trên đối xứng qua các cạnh tương ứng ta được 3 điểm nữa là và .Khi ấy tam giác được gọi là tam giác của tam giác. Đường tròn ngoại tiếp tam giác. .. được gọi là đường tròn. Tính chất: 1). S=−. ( ∑ a3 − ∑ a2 b+3 abc) S ABC ( a+b+ c)OI2 =− (b+ c − a)( a+c −b)( a+b −c ) 4R. 2). 3)Trực tâm của tam giác. trùng với tâm đường tròn nội tiếp của tam giác. ..

<span class='text_page_counter'>(139)</span> (để chứng minh ta có thế sử dụng tích vô hướng) 4)Tâm đường tròn chín điểm của tam giác đường tròn ngoại tiếp tam giác. và tam giác có độ dài bằng. trùng nhau. Bán kính. II.25)Hình luc giác và đường tròn Lemoine thứ nhất Kết quả:Cho tam giác và điểm các cạnh cắt các cạnh còn lại tại giác thứ nhất của tam giác. . Qua kẻ các đường thẳng song song với . Khi ấy lục giác được gọi là lục .. Tính chất: 1)Lục giác thứ nhất là lục giác ngoại tiếp. Đường tròn ngoại tiếp lục giác này được gọi là đường tròn thứ nhất: Do là đường thẳng đối trung đi qua trung điểm của nên là đường đối song tương ứng cạnh . Suy ra tứ giác là tứ giác nội tiếp. 2)Các cạnh bị kẹp giữa các đường song song tỉ lệ với lũy thừa bậc ba cạnh tương ứng: Gọi x, y, z là khoảng cách từ L tới 3 cạnh tam giác. Khi đó:. 3)3 đoạn trên cùng 1 cạnh tỉ lệ với bình phương các cạnh của tam giác.

<span class='text_page_counter'>(140)</span> 4)Tâm của đường tròn Lemoine thứ nhất là trung điểm đoạn nối điểm Lemoine với tâm đường tròn ngoại tiếp tam giác ABC: Để ý vuông góc với khi đó ta dễ dàng suy ra được dpcm. 5)Bán kính Ngoài ra ta còn có cách viết khác: Trong đó. là bán kính đường tròn Lemoine thứ hai(sẽ được nói đến trong phần sau). II.26)Hình lục giác và đường tròn Lemoine thứ hai Khái niệm về đường đối song:Cho tam giác . Với 2 điểm bất kì thuộc các cạnh và ta có 2 kiểu chọn sao cho tam giác đồng dạng với tam giác . Thứ nhất là song song với Thứ hai là tứ giác nội tiếp như hình vẽ. Khi ấy và được gọi là các đường đối song tương ứng với góc Tính chất:Đường đối trung luôn đi qua trung điểm của các đường đối song tương ứng với cùng 1 đỉnh.. Kết quả:Cho tam giác và điểm . Qua với các cạnh của tam giác cắt các cạnh còn lại tại được gọi là lục giác thứ hai.. kẻ các đường đối song tương ứng Khi đó lục giác.

<span class='text_page_counter'>(141)</span> Tính chất: 1)Lục giác. thứ hai nội tiếp 1 đường tròn và đường tròn này được gọi là đường tròn thứ hai của tam giác ABC.. 2)Bán kính Thực chất lục giác và đường tròn. và đường tròn . Xem thêm trong. chỉ là trường hợp đặc biệt của lục giác. II.27)Điểm Euler của Tứ giác nội tiếp Kết quả:Cho tứ giác nội tiếp . Gọi lần lượt là trực tâm các tam giác và . Khi ấy thì các đường thẳng và đồng quy. Điểm đồng quy được gọi là điểm của tứ giác nội tiếp..

<span class='text_page_counter'>(142)</span> Chỉ dẫn chứng minh: Ta có song song và cùng bằng 2 lần khoảng cách từ tới nên tứ giác là hình bình hành suy ra giao nhau tại trung điểm mỗi đường. Tương tự ta suy ra bốn đường thẳng đồng quy tại trung điểm mỗi đường. Từ đây ta suy ra được nhiều tính chất thú vị của điểm 1)Điểm nằm trên đường vuông góc hạ từ trung điểm một cạnh tới cạnh đối diện(hoặc trung điểm đường chéo tới đường chéo còn lại). 2)Đường thẳng các đỉnh còn lại.. của đỉnh. với tam giác. 3)Đường tròn chín điểm của các tam giác. thì đi qua điểm. và. . Tương tự với. đồng quy tại điểm. Xem thêm. II.28)Đường thẳng Steiner của tứ giác toàn phần. Kết quả:Cho tứ giác toàn phần Khi đó trực tâm của các tam giác và cùng nằm trên 1 đường thẳng được gọi là đường thẳng của tứ giác toàn phần..

<span class='text_page_counter'>(143)</span> Chỉ dẫn chứng minh: Gọi lần lượt là trực tâm các tam giác Gọi là trung điểm các đường chéo Khi đó:. và. Vậy nằm trên trục đẳng phương của 2 đường tròn và Tương tự ta cũng có 3 trực tâm còn lại cùng nằm trên trục đẳng phương của hai đường tròn này suy ra dpcm.. II.29)Đường thẳng Gauss của tứ giác toàn phần. Kết quả:Cho tư giác toàn phần một đường thẳng được gọi là đường thẳng. . Khi đó trung điểm các đường chéo cùng nằm trên của tứ giác toàn phần..

<span class='text_page_counter'>(144)</span> Chỉ dẫn chứng minh: Gọi lần lượt là trung điểm các đường chéo là tam giác trung bình của tam giác Khi đó các điểm nằm trên các cạnh của tam giác Ta có:. và .. Nhân các vế các đẳng thức trên ta được:. MK NI PJ EA FB DC . . = . . =1 MI NJ PK EC FA DB. Suy ra dpcm. Từ 2 bài viết trên ta thấy rằng trong 1 tứ giác toàn phần thì đường thẳng Steiner vuông góc với đường thẳng Gauss.. II.30) Điểm Miquel của tứ giác toàn phần Kết quả:Cho tứ giác toàn phần . Khi ấy đường tròn ngoại tiếp của các tam giác và đồng quy. Điểm đồng quy đó được gọi là điểm của tứ giác toàn phần..

<span class='text_page_counter'>(145)</span> Chỉ dẫn chứng minh: Giả sử đường tròn ngoại tiếp các tam giác Khi đó ta có:. và. giao nhau tại điểm. khác. (⃗ MA ; ⃗ MC)≡(⃗ MA ; ⃗ ME)+(⃗ ME ; ⃗ MC)≡(− ⃗ FA ; ⃗ FE)+( ⃗ DE ; ⃗ DC)≡( ⃗ AF ; ⃗ AE)+(⃗ AE ; ⃗ FE)+(⃗ DE ; ⃗ DC)≡ (⃗ AF ; ⃗ AE)+(⃗ EC ; ⃗ ED)+(⃗ DE ; ⃗ DC) ≡(⃗ AB ; ⃗ AC)+(⃗ CA ; ⃗ CB)≡ π −(⃗ BC ; ⃗ BA )≡(⃗ AB ; ⃗ BC) mod 2 π Vậy nằm trên đường tròn ngoại tiếp tam giác ra dpcm.. . Tương tự với đường tròn còn lại ta suy. II.31)Đường tròn Miquel của tứ giác toàn phần Kết quả:Cho tứ giác toàn phần ngoại tiếp tam giác của tứ giác toàn phần.. và. Khi đó điểm và tâm của các đường tròn cùng nằm trên 1 đường tròn - đường tròn.

<span class='text_page_counter'>(146)</span> Chỉ dẫn chứng minh: Gọi và lần lượt là tâm đường tròn ngoại tiếp các tam giác . Gọi lần lượt là chân các đường vuông góc kẻ từ tới và Do là trung điểm của nên chúng thẳng hàng. Theo định lí đảo về đường thẳng Simson ta có cùng nằm trên 1 đường tròn. Tương tự ta suy ra dpcm.. và. (Thêm phần tứ giác toàn phần kèm theo, File BDF). II.32)Hình bình hành Varignon của tứ giác . Kết quả:Cho tứ giác ABCD có M,N,P,Q lần lượt là trung điểm của AB,BC,CD,DA.Khi đó M,N,P,Q là bốn đỉnh của một hình bình hành gọi là hình bình hành Varignon của tứ giác ABCD. Chỉ dẫn chứng minh:.

<span class='text_page_counter'>(147)</span> Chứng minh kết quả này khá đơn giản,dễ thấy MN,PQ tương ứng là đường trung bình của các tam giác ABC và ACD thế nên Do vậy MNPQ là một hình bình hành.. .. I.71)Định lí Poncelet về bán kính đường tròn nội tiếp,bàng tiếp trong tam giác vuông. Định lí:Cho tam giác có lần lượt là bán kính các đường tròn nội tiếp, bàng tiếp góc .Chứng minh rằng: tam giác ABC vuông tại A khi và chỉ khi .. Chứng minh:: Ta có:. tam giác. vuông tại. (dpcm).

<span class='text_page_counter'>(148)</span>

Tài liệu bạn tìm kiếm đã sẵn sàng tải về

Tải bản đầy đủ ngay
×